playmath 16-17

100
ˇ CASOPIS JE POKRENUO 3. F. RAZRED ˇ SKOLSKE GODINE 2002./2003. πlαy matχ PlayMath ˇ casopis za matematiku i informatiku ISSN 1334-7586 broj 16-17 godiˇ ste VI 8. listopada 2008. I ZDAVA ˇ CI : V. gimnazija, Klai´ ceva 1, Zagreb Hrvatsko matematiˇ cko druˇ stvo, Bijeniˇ cka 30, Zagreb ˇ CASOPIS IZLAZI TRI PUTA TIJEKOM ˇ SKOLSKE GODINE UREDNI ˇ STVO: UREDNICI RUBRIKA: Mislav Mijatovi´ c Viktorija Sukser Grgur Valenti´ c Ivan Gavran Karlo Dumbovi´ c Marin Buˇ zanˇ ci´ c Zrinka Gavran Ines Maruˇ si´ c GRAFI ˇ CKI UREDNIK: UREDNIK WEB- IZDANJA: Vilim ˇ Stih Tomislav Gudlek UREDNI ˇ CKI ODBOR: Ivan Radoˇ sevi´ c Goran ˇ Zuˇ zi´ c Ivo Sluganovi´ c Frane Kurtovi´ c Marina Sliˇ skovi´ c Hrvoje Stojanovi´ c Borna Vukadinovi´ c Matija Baˇ si´ c GLAVNI UREDNIK: Adrian Satja Kurdija ([email protected]) STRU ˇ CNI SURADNICI: RECENZENTICA: LEKTORICA ZA HRVATSKI JEZIK: Jelena Gusi´ c, prof. Smiljana Karluˇ si´ c Koˇ zar, prof. AUTOR ORNAMENATA: STRU ˇ CNI SAVJETNIK: prof. dr. sc. Darko ˇ Zubrini´ c prof. dr. Zvonimir ˇ Siki´ c TEHNI ˇ CKI SAVJETNICI : Tvrtko Tadi´ c, dipl. inˇ z. (tiskano izdanje) Filip Nikˇ si´ c (web-izdanje) ZA IZDAVA ˇ CE: Petar Mladini´ c, prof. ADRESA ˇ CASOPISA: UREDNI ˇ STVO: V. gimnazija (PlayMath) Klai´ ceva 1 10000 Zagreb [email protected] NARUD ˇ ZBE: HMD Bijeniˇ cka 30 p.p. 335 10002 Zagreb [email protected] URL: http://playmath.petagimnazija.hr ˇ Casopis izlazi uz pomo´ c Ministarstva znanosti, obrazovanja i ˇ sporta RH PlayMath je ukljuˇ cen u Portal znanstvenih ˇ casopisa RH - Hrˇ cak http://hrcak.srce.hr Tisak: Tiskara Zelina d.d., Sveti Ivan Zelina

Upload: domagoj-pluscec

Post on 04-Jan-2016

107 views

Category:

Documents


4 download

DESCRIPTION

playmath

TRANSCRIPT

Page 1: PlayMath 16-17

CASOPIS JE POKRENUO 3.F. RAZRED SKOLSKE GODINE 2002./2003.���������������������������������������������������������πlαy√

matχPlayMath

casopis za matematiku i informatiku�������������������ISSN 1334-7586

������������������broj 16-17godiste VI

8. listopada 2008.IZDAVACI:

V. gimnazija, Klaiceva 1, ZagrebHrvatsko matematicko drustvo, Bijenicka 30, Zagreb��������������������������������������������������������

CASOPIS IZLAZI TRI PUTA TIJEKOM SKOLSKE GODINE

UREDNISTVO:

UREDNICI RUBRIKA:

Mislav Mijatovic Viktorija SukserGrgur Valentic Ivan Gavran

Karlo Dumbovic Marin BuzancicZrinka Gavran Ines Marusic

GRAFICKI UREDNIK: UREDNIK WEB-IZDANJA:Vilim Stih Tomislav Gudlek

UREDNICKI ODBOR:Ivan Radosevic Goran Zuzic Ivo SluganovicFrane Kurtovic Marina Sliskovic Hrvoje Stojanovic

Borna Vukadinovic Matija BasicGLAVNI UREDNIK:

Adrian Satja Kurdija ([email protected])

STRUCNI SURADNICI:RECENZENTICA: LEKTORICA ZA HRVATSKI JEZIK:Jelena Gusic, prof. Smiljana Karlusic Kozar, prof.

AUTOR ORNAMENATA: STRUCNI SAVJETNIK:prof. dr. sc. Darko Zubrinic prof. dr. Zvonimir Sikic

TEHNICKI SAVJETNICI:Tvrtko Tadic, dipl. inz. (tiskano izdanje) Filip Niksic (web-izdanje)

ZA IZDAVACE:Petar Mladinic, prof.

ADRESA CASOPISA:UREDNISTVO:V. gimnazija(PlayMath)Klaiceva 1

10000 [email protected]

NARUDZBE:HMD

Bijenicka 30p.p. 335

10002 [email protected]

URL:http://playmath.petagimnazija.hr

Casopis izlazi uz pomoc Ministarstva znanosti, obrazovanja i sporta RHPlayMath je ukljucen u Portal znanstvenih casopisa RH - Hrcak http://hrcak.srce.hr���������������������������������������������������������

Tisak: Tiskara Zelina d.d., Sveti Ivan Zelina

Page 2: PlayMath 16-17

THE MAGAZINE WAS STARTED BY 11TH F CLASS IN THE SCHOOL YEAR 2002/2003.���������������������������������������������������������πlαy√

matχPlayMath

magazine for mathematics and computer science�������������������ISSN 1334-7586

������������������number 16-17

volume VIOctober 2008PUBLISHERS:

5th High School, Klaiceva 1, ZagrebCroatian Mathematical Society, Bijenicka 30, Zagreb��������������������������������������������������������

THE MAGAZINE IS PUBLISHED THREE TIMES A YEAR

EDITORIAL BOARD:SECTION EDITORS:

Mislav Mijatovic Viktorija SukserGrgur Valentic Ivan Gavran

Karlo Dumbovic Marin BuzancicZrinka Gavran Ines Marusic

GRAPHICAL EDITOR: WEB EDITION EDITOR:Vilim Stih Tomislav Gudlek

EDITORIAL COMMITTEE:Ivan Radosevic Goran Zuzic Ivo SluganovicFrane Kurtovic Marina Sliskovic Hrvoje Stojanovic

Borna Vukadinovic Matija BasicEDITOR-IN-CHIEF:

Adrian Satja Kurdija ([email protected])

ADVISORY BOARD:REVIEWER: ENGLISH LANGUAGE LECTORS:

Jelena Gusic Sanja Vrhovec VucemilovicMilena Gilic

TEACHER, 15th HIGH SCHOOL TEACHERS, 5th HIGH SCHOOL

ORNAMENTS AUTHOR: ADVISOR:Darko Zubrinic Zvonimir Sikic

PROFESSOR, UNIVERSITY OF ZAGREB PROFESSOR, UNIVERSITY OF ZAGREB

TECHNICAL ADVISORS:Tvrtko Tadic (print edition) Filip Niksic (web edition)

PUBLISHER:Petar Mladinic

PRINCIPAL, 5th HIGH SCHOOL & VICE-PRESIDENT, CROATIAN MATHEMATICAL SOCIETY

ADDRESS:EDITORIAL BOARD:

V. gimnazija(PlayMath)Klaiceva 1

10000 [email protected]

SUBSCRIPTION INQUIRIES:HMD

Bijenicka 30p.p. 335

10002 [email protected]

URL:http://playmath.petagimnazija.hr

The magazine is published with the assistance of the Ministry of Science, Education and Sports of the Republic of CroatiaPlayMath is listed on the Portal of Scientific Magazines of the Republic of Croatia - Hrcak http://hrcak.srce.hr���������������������������������������������������������

Printed by Tiskara Zelina d.d., Sveti Ivan Zelina, Croatia

Page 3: PlayMath 16-17

PlayMath je casopisnamijenjen mladimmatematicarima prven-stveno srednjoskolcima.Urednistvo cine ucenicii studenti. U casopisu seobjavljuju clanci, prilozi,vijesti, zabavni sadrzaji(vicevi, anegdote. . . ),zadatci i rjesenja, ilus-tracije i ostali materijalivezani uz matematiku (iinformatiku). Jedan odglavnih ciljeva casopisaje razmjena ideja iiskustava. Svi citatelji(ucenici i studenti)su pozvani da posaljuclanak, prilog, zadatak,salu, anegdotu ili nestoslicno sto bi moglobiti zanimljivo ostalimcitateljima, a spada uprofil casopisa. Clancineka ne budu dulji od5 stranica. Sve prilogemozete poslati na e-mailili na postansku adresucasopisa.

Sadrzaj70 godina V. gimnazije i matematike u njoj . . . . . . . . . . . . . . . . . . . . 5

ClanciSuradnja ucenika V. gimnazije s matematickim casopisima . . . . . . . . . . . 6Tomislav Pejkovic: Kako sam pomogao Matku . . . . . . . . . . . . . . . . . . 7Adrian Satja Kurdija: Na koliko nacina? . . . . . . . . . . . . . . . . . . . . . 11Tvrtko Tadic: Jednake znamenke = potencija . . . . . . . . . . . . . . . . . . . 15

Matematicka igraonicaMarko Horvat, Filip Niksic: Igra sest stotica . . . . . . . . . . . . . . . . . . . 18

Veliki matematicariRudi Mrazovic: Srinivasa Ramanujan . . . . . . . . . . . . . . . . . . . . . . . 22

Matematicki softwareMatematika pomocu racunala . . . . . . . . . . . . . . . . . . . . . . . . . . . 25Raketa i avion . . . . . . . . . . . . . . . . . . . . . . . . . . . . . . . . . . . 26Suncani sat . . . . . . . . . . . . . . . . . . . . . . . . . . . . . . . . . . . . . 29Dinamicni model Ptolomejevog sustava . . . . . . . . . . . . . . . . . . . . . . 31O jednom preslikavanju . . . . . . . . . . . . . . . . . . . . . . . . . . . . . . 35

Natjecanja u V. gimnazijiRepublicko natjecanje 1960./1961. . . . . . . . . . . . . . . . . . . . . . . . . 44Republicko natjecanje 1964./1965. . . . . . . . . . . . . . . . . . . . . . . . . 46Savezno natjecanje 1967./1968. . . . . . . . . . . . . . . . . . . . . . . . . . . 50Savezno natjecanje 1968./1969. . . . . . . . . . . . . . . . . . . . . . . . . . . 53Olimpijci V. gimnazije iz matematike . . . . . . . . . . . . . . . . . . . . . . . 55Grupe za natjecanja u V. gimnaziji . . . . . . . . . . . . . . . . . . . . . . . . 61

Zadatci i rjesenjaOdredi broj! . . . . . . . . . . . . . . . . . . . . . . . . . . . . . . . . . . . . 62Tri rjesenja jedne nejednakosti . . . . . . . . . . . . . . . . . . . . . . . . . . 63Dinamicko preslagivanje trokuta . . . . . . . . . . . . . . . . . . . . . . . . . 65Cetiri kruznice i njihova zajednicka tocka . . . . . . . . . . . . . . . . . . . . . 68Polinomi i prosti brojevi . . . . . . . . . . . . . . . . . . . . . . . . . . . . . . 69Upute i rjesenja . . . . . . . . . . . . . . . . . . . . . . . . . . . . . . . . . . 70

PovijestPlayMath . . . . . . . . . . . . . . . . . . . . . . . . . . . . . . . . . . . . . . 96

Page 4: PlayMath 16-17

PlayMath is a magazinefor young mathematici-ans. The editors arehigh school and univer-sity students. The ma-gazine publishes articles,notes, news, fun ma-terial (jokes, interestingstories . . . ), problemsand solutions, illustrati-ons and other materialsin high school mathema-tics and computer sci-ence. The main goal ofthe magazine is coopera-tion among students. Allreaders (high school anduniversity students) areinvited to contribute tothe magazine. Materialsshould be up to 5 pages.All materials can be sentto our e-mail address orto our mailing address.

In this Issue70 Years of the 5th High School Zagreb and Math in It . . . . . . . . . . . 5

ArticlesStudents and Mathematical Magazines . . . . . . . . . . . . . . . . . . . 6Tomislav Pejkovic: How I Helped Matko . . . . . . . . . . . . . . . . . . 7Adrian Satja Kurdija: In How Many Ways? . . . . . . . . . . . . . . . . 11Tvrtko Tadic: Equal Digits = Power . . . . . . . . . . . . . . . . . . . . . 15

Mathematical PlaygroundMarko Horvat, Filip Niksic: The Game of Six Hundreds . . . . . . . . . . 18

Great MathematiciansRudi Mrazovic: Srinivasa Ramanujan . . . . . . . . . . . . . . . . . . . . 22

Mathematical SoftwareMathematics with the Computer Help . . . . . . . . . . . . . . . . . . . . 25Rocket and an Airplane . . . . . . . . . . . . . . . . . . . . . . . . . . . 26Shadow Clock . . . . . . . . . . . . . . . . . . . . . . . . . . . . . . . . 29Dynamical Model of the Ptolomej System . . . . . . . . . . . . . . . . . 31About a Transformation . . . . . . . . . . . . . . . . . . . . . . . . . . . 35

Competitions in the 5th High SchoolThe Republic Competition 1960/1961 . . . . . . . . . . . . . . . . . . . . 44The Republic Competition 1964/1965 . . . . . . . . . . . . . . . . . . . . 46The Federal Competition 1967/1968 . . . . . . . . . . . . . . . . . . . . 50The Federal Competition 1968/1969 . . . . . . . . . . . . . . . . . . . . 53IMO participants form the 5th High School . . . . . . . . . . . . . . . . . 58Math Circles in the 5th High School . . . . . . . . . . . . . . . . . . . . 61

Problems and SolutionsDetermine the Number! . . . . . . . . . . . . . . . . . . . . . . . . . . . 62Three Solutions of an Inequality . . . . . . . . . . . . . . . . . . . . . . . 63Dynamical Reshuffle of a Triangle . . . . . . . . . . . . . . . . . . . . . 65Four Circles and a Common Point . . . . . . . . . . . . . . . . . . . . . . 68Polynomials and Prime Numbers . . . . . . . . . . . . . . . . . . . . . . 69Hints and Solutions . . . . . . . . . . . . . . . . . . . . . . . . . . . . . 70

HistoryPlayMath . . . . . . . . . . . . . . . . . . . . . . . . . . . . . . . . . . . 96

Page 5: PlayMath 16-17

πlαy√

matχ

70 godina V. gimnazije i matematike u njoj

Dragi citatelji, 2008. godine obiljezavamo 70 godina djelovanja V. gimnazije u Zagrebu.

V. gimnazija je postale prepoznatljiva po rezultatima svojih mladih matematicara, od kojih su mnogi kasnijepostali poznati i ugledni matematicari, a dobar dio je stekao ugled i drugim (srodnim) podrucjima (fizika,elektrotehnika, racunarstvo. . . ).

Kako bi se prikljucili obiljezavanju ove obljetnice urednistvo PlayMath-a odlucilo je istraziti kako je izgle-dao rad s mladim matematicarima u V. gimnaziji u tom razdoblju, koje su rezultate postizali, tko im je u tomepomagao i gdje su danas ti ljudi.

Koncepcija broja

Ovaj broj je zamisljen da bude kao svaki drugi broj PlayMath-a s time da ce u ovom broju objavljenimaterijali biti prigodni.

Clanci u ovom broju su oni koje su ucenici V. gimnazije objavili u raznim casopisima (prije pojavljivanjaPlayMath-a), odabrani zadatci ce sadrzavati rjesenja koja su ucenici dali u raznim prigodama, bit ce objavljenaneka stara natjecanja (s rjesenjima) na kojima su ucenici V. gimnazije imali poseban uspjeh. . .

Svim buducim i sadasnjim ucenicima V. gimnazije zelimo puno uspjeha u daljem radu.Urednistvo

70 Years of the 5th High School Zagreb and Math in It

Dear readers, this year we are celebrating the 70th anniversary of the 5th High School (for math andscience) in Zagreb. PlayMath as its publication for mathematics and computer science is going to rememberthe students and teachers, competition’s results and articles published by students in (student) math journals.

This special edition of PlayMath has been partly translated into English. Table of contents, article abstracts,results of competitions, and some texts are in English.

The 5th High School in known for its student’s results on national and international competitions. Many ofits students later became distinguished mathematicians, and some of them are known for their result in other(related) areas (such as physics, computer science, biology, electrotechnics).

This Issue

This (two-number) Issue is intended to celebrate the occasion and look like all the regular Issues (witharticles, problems, reviews . . . ).

We have prepared articles written by the students of the 5th high school for other magazines during theseyears, old competitions, competition results, selected problems with special solutions given by the students,reviews written by the students, the overlook of the student activities . . .

Editors

http://playmath.petagimnazija.hr 5

Page 6: PlayMath 16-17

πlαy√

matχ

Suradnja ucenika V. gimnazije s matematickimcasopisima

Ucenici V. gimnazije zainteresirani za matematiku vec dugi niz godina (vise od pola stoljeca) suraduju sraznim casopisima - salju rjesenja zadataka postavljenih u njima, pisu clanke, salju zadatke, a od osnivanjaPlayMath-a rade i kao urednici.

Ovakav oblik izvannastavnog rada ima nekoliko korisnih pojava:

• Ucenici dobivaju dragocjeno iskustvo (posebno kod pisanja clanaka i izvjestaja) u komuniciranju s kole-gama i strucnom (cesto i sirom) javnoscu svojih kolega.

• Stjecu radno iskustvo kroz rad, postivanje rokova. . .

• Vide rezultate svoga rada (u objavljenim clancima, rjesenjima, izvjestajima, . . . ) i imaju priliku cutikomentare.

• . . .

U nekoliko clanaka koji slijede odabrali smo (medu clancima koji su nam bili dostupni) nekoliko posebnihclanaka koje su objavili ucenici V. gimnazije. �

Students and Mathematical Magazines

Students form the 5th High School who are interested in mathematics have been cooperating with mathema-tical magazines (for students) in Croatia for more then half a century. They wrote solutions to posed problems,articles on different topics and posed problems. And since the start of PlayMath they have been working aseditors.

All students included in this type of work gained valuable experience. They have experienced communica-tion with the fellow-students with the similar interests and the mathematical community (and also often withgeneral public).

The articles that follow are specially chosen ones (from those available). They are the 5th High Schoolstudents’ work. �

6 PlayMath br. 16-17 (2008.)

Page 7: PlayMath 16-17

πlαy√

matχ

Kako sam pomogao Matku

Tomislav Pejkovic

Clanak je izvorno objavljen u casopisu Matka br. 31 (2000.).

Matko je zelio izracunati potenciju1 tezista trokuta s obzirom na opisanu kruznicu tog trokuta. Potencijatezista treba biti izrazena pomocu duljina stranica tog trokuta.

Rjesenje. Neka je u trokutu ABC tocka T teziste, A′ i B′ redom polovista stranica BC i AC . Neka je Kpresjek pravaca AA′ i kruznice opisane trokutu ABC .

Slika 1.

Duljine stranica tog trokuta ABC oznacimo s a, b i c, i neka je |AA′| = ta. Pronadimo najprije potencijutocke A′ s obzirom na opisanu kruznicu. Vrijedi:

|BA′| · |A′C| = |AA′| · |A′K|,

tj. |A′K| =a2

4ta.

Buduci da teziste dijeli tezisnicu u omjeru 2 : 1 (gledano od vrha trokuta), imamo:

|AT | =2

3ta, |TA′| =

1

3ta,

1Vidi prvi dodatak iza clanka.

http://playmath.petagimnazija.hr 7

Page 8: PlayMath 16-17

πlαy√

matχpa je potencija tocke T s obzirom na danu kruznicu:

|AT | · |TK| = |AT |(|TA′| + |A′K|)

=2

3ta ·(

1

3ta +

a2

4ta

)

=2

9t2a +

a2

6.

Duljinu tezisnice trokuta mozemo izraziti preko duljina stranica rabeci relaciju paralelograma2 . Dobivamopoznatu relaciju

4t2a = 2b2 + 2c2 − a2.

Uvrstavanjem konacno dobivamo rjesenje Matkova problema. Potencija tezista T s obzirom na kruznicu opi-sanu trokutu ABC je:

a2 + b2 + c2

9.

X

Potencija tocke s obzirom na kruznicu

Neka je k kruznica. Ako je tocka T izvan kruznice k, tada se potencija tocke T u odnosu nakruznicu k definira kao umnozak:

|TA| · |TB|,gdje su A, B tocke u kojima neki pravac, koji prolazi kroz T , sijece kruznicu k. Umnozak ne ovisio izboru pravca. (U slucaju kao na prvoj slici vrijedi: |TA||TB| = |TA′||TB′|.)

Ako je tocka T unutar kruznice k i A, B su tocke u kojima neki pravac, koji prolazi kroz T ,sijece kruznicu k. Tada se, na isti nacin, potencija tocke T u odnosu na kruznicu k definira kaoumnozak

|TA| · |TB|.(U slucaju kao na drugoj slici vrijedi |TA||TB| = |TA′||TB′|.)

2Vidi drugi dodatak iza clanka.

8 PlayMath br. 16-17 (2008.)

Page 9: PlayMath 16-17

πlαy√

matχRelacija paralelograma

Neka je ABCD paralelogram. Tada vrijedi:

|AB|2 + |BC|2 + |CD|2 + |DA|2 = |AC|2 + |BD|2,

odnosno, oznacimo li a = |AB| = |CD|, b = |BC| = |DA|, e = |AC| i f = |BD|, vrijedi:

e2 + f2 = 2a2 + 2b2.

Trokut ABC dopunimo do paralelograma ABA′C , gdje je tocka A′ dobivena centralnom sime-trijom tocke A u odnosu na poloviste Pa stranice BC. Iz relacije paralelograma lako dobivamo daje

t2a =1

4(2b2 + 2c2 − a2).

O autoru

Tomislav Pejkovic dobitnik je dvije broncane medalje na Medunarodnim matematickim olimpijadama 1999.i 2000. godine. Kao promatrac sudjelovao je na MMO 1998. Autor je knjige Iracionalni brojevi iz Matkinebiblioteke.

Danas je asistent na PMF-Matematickom odjelu (ZAVOD ZA ALGEBRU I OSNOVE MATEMATIKE) u Za-grebu. Zaposlen je na znanstvenom projektu Diofantske jednadzbe kojeg vodi prof. dr. sc. Andrej Dujella.Clan je Drzavnog povjerenstva za matematicka natjecanja i Izvrsnog odbora Hrvatskog matematickog drustva.

Abstract

In this article the author explores a way to calculate the power of a point for the centroid of the triangle regardingits circumscribed circle.

About the author

Tomislav Pejkovic won two third prizes at the International Mathematical Olympiads in 1999 and 2000. Heparticipated as an observer at the IMO in 1998. He is the author of the book Iracionalni brojevi (IrrationalNumbers) published by the Croatian Mathematical Society.

Today, Tomislav is a teaching/research assistant at the Department of Mathematics University of Zagreb(Division for Algebra and Foundations of Mathematics). He works on the project Diophantine Equationsheaded by professor Andrej Dujella. Tomislav is a member of the National Mathematics Competition Boardand a member of the Executive Board of the Croatian Mathematical Society.

http://playmath.petagimnazija.hr 9

Page 10: PlayMath 16-17

πlαy√

matχ

10 PlayMath br. 16-17 (2008.)

Page 11: PlayMath 16-17

πlαy√

matχ

Na koliko nacina?

Adrian Satja KurdijaClanak je preuzet iz Matke br. 60 (2007.).

Nagradni zadatak broj 54 u Matki je glasio:Na slici je tlocrt gradske cevrti u Matkogradu. Na koliko razlicitih nacina mozete doci iz A u C?Mi cemo sliku zadatka, radi jednostavnosti, malo izmijeniti. Grad cemo nacrtati kao mrezu ulica, cije su

dimenzije 4 × 4:

Slika 1.

Pokusajmo napisati program u Pascalu koji rjesava ovaj problem za grad dimenzija m × n, a ne samo za4 × 4. Algoritam kojeg sam ja koristio je sljedeci: na svako raskrizje, iduci redom od pocetne tocke (0, 0),upisuje se broj nacina na koje se do tog raskrizja moze doci, sve dok ne dodemo do konacne tocke (m,n). Najedno raskrizje moguce je doci jedino preko raskrizja neposredno ispod i neposredno lijevo od njega, pa je brojnacina za dolazak na njega jednak zbroju dvaju brojeva na tim dvama raskrizjima.

Tok algoritma je sljedeci. Svaki korak bit ce popracen primjerom za tablicu 4 × 4.

– najprije raskrizjima koji se nalaze na nultoj horizontalnoj ili vertikalnoj liniji mreze dodjeljujem vrijed-nost 1, jer je do svakog od tih raskrizja moguce doci na samo jedan nacin.

Slika 2.

http://playmath.petagimnazija.hr 11

Page 12: PlayMath 16-17

πlαy√

matχ– zatim, iduci po redovima, s lijeva na desno dodjeljujem vrijednosti raskrizjima po opisanom postupku:

svako raskrizje poprima vrijednost zbroja brojeva na raskrizjima neposredno ispod i neposredno lijevood njega.

Slika 3.

– konacno, iz popunjene tablice ocitava se vrijednost posljednjeg polja (u nasem slucaju, to je 70).

Slika 4.

A sada slijedi kod u Pascalu:

program matkograd;

type raskrizja = array [0..100, 0..100] of integer;

var r : raskrizja;m, n, i, j : integer;

beginreadln (m, n);

for i:=0 to m dor[i, 0] :=1;

for i:=0 to n do

12 PlayMath br. 16-17 (2008.)

Page 13: PlayMath 16-17

πlαy√

matχr[0, i] :=1;

for i:=1 to m do for j:=1 to n dor[i,j] := r[i, j-1] + r[i-1, j];

writeln ( r[m, n] );

readln;end.

Medutim, algoritam za nalazenje rjesenja pravog matematicara nece zadovoljiti. Trazimo formulu za nalazenjebroja nacina u opcem slucaju m×n. Ako isprobamo nas program za razne slucajeve, jos uvijek necemo naslu-titi formulu buduci da ona i nije bas jednostavna. To nas upucuje na sustavno razmatranje problema. Krenimona posao!

Odsada, ”stupcem” cemo zvati svaku od m+1 vertikalnih linija tablice, a ”retkom” svaku od n+1 horizon-talnih linija tablice. I retke i stupce oznacit cemo sa 0, 1, . . . , n (odnosno m), kao sto smo to cinili u programu.Pojednostavljujuci problem, dovoljno je promatrati samo vertikalne jedinicne linije kretanja (nazovimo ih v-linije) buduci da je onda lako docrtati i horizontalne jedinicne linije kretanja (nazovimo ih h-linije). Lako jezakljuciti da se svaki trazeni put sastoji od n v-linija i m h-linija.

Dokazimo sada sljedecu tvrdnju: rasporedimo li v-linije tako da odredimo koliko se njih nalazi u nultom,koliko u prvom, i tako dalje..., koliko u m-tom stupcu, pri cemu je ukupno n v-linija, tada svakom takvomrasporedu odgovara tocno jedan trazeni put.

Dokaz. Buduci da se pri kretanju nije moguce vracati, v-linije su poredane uzlazno, sto znaci da, ako je x < y,nije moguce da v-linija u y-tom stupcu bude nize od v-linije u x-tom stupcu. Buduci da svakoj od n razinatablice odgovara tocno jedna v-linija, za bilo koji zadani raspored dobivamo jedinstven trazeni put na sljedecinacin. Idemo po redu, od nultog stupca, crtajuci v-linije od najnize razine, onoliko koliko ih tamo ima. Nas-tavljajuci postupak, opcenito, u svakom stupcu crtamo onoliko v-linija koliko je zadano redom prema gore,polazeci od najdonje razine na kojoj jos nije nacrtana v-linija. Na taj nacin dobivamo jedinstveno rasporedenev-linije, a na njih je lako docrtati h-linije, sto daje trazeni put. �

Dakle, zadatak mozemo preformulirati ovako: na koliko nacina je moguce n v-linija rasporediti u m + 1stupaca, pri cemu mogu postojati stupci koji ne sadrze niti jednu v-liniju? Ili, ekvivalentno, na koliko nacina jemoguce n jednakih knjiga rasporediti na m + 1 razlicitih polica?

Ovaj problem zahtijeva i malo znanja kombinatorike. Primijetimo da svakom rasporedu knjiga na policeodgovara jedinstvena rijec sastavljena od n slova K i m slova P , pri cemu vrijedi i obrat, gdje K oznacavaknjigu a P oznacava ”pregradu” izmedu dviju polica. Na primjer, KKPKPPK bi znacilo da se na prvoj policinalaze 2 knjige, na drugoj jedna, na trecoj niti jedna i na cetvrtoj jedna knjiga. Problem je zato ekvivalentansljedecem: Na koliko nacina je u rijeci od m+n slova K i P moguce odabrati pozicije n slova K? (Ili m slovaP , rezultat je isti). Ovaj broj je jednak:

(m + n

n

)

=

(m + n

m

)

=(m + n)!

m!n!

U slucaju iz naseg zadatka, gdje je m = n = 4, ovaj broj je jednak 70. Takoder, istu bismo formulu dobilida smo, umjesto v-linija, promatrali raspored h-linija.

http://playmath.petagimnazija.hr 13

Page 14: PlayMath 16-17

πlαy√

matχO autoru

Adrian Satja Kurdija trenutni je glavni urednik PlayMath-a (od 2007. godine). Dobitnik je brojnih nagrada nadrzavnim natjecanjima iz matematike te BRONCANE MEDALJE na ovogodisnjoj Medunarodnoj matematickojolimpijadi.

Abstract

The article presents a solution to the Prize problem posed by magazine Matka no. 54. This article was publishedin Matka no. 60.

About the author

Adrian Satja Kurdija is the Editor-in-Chief of PlayMath. He won several prizes at the National MathematicsCompetitions and a bronze medal at this year’s International Mathematical Olympiad.

14 PlayMath br. 16-17 (2008.)

Page 15: PlayMath 16-17

πlαy√

matχ

Jednake znamenke = potencija

Tvrtko Tadic

Ovaj clanak objavljen je u casopisu Matka br. 44 (2003.). Prenosimo clanak u cijelosti.

Mnoge ce naslov zacuditi1 pa nece niti poceti citati clanak. No oni koji se odluce na citanje – ne trebajuse bojati. Naime, u clanku se pojavljuju iskljucivo potencije broja 10, i to samo kao oznake. Kako bi nas sviMatkaci (pa i oni koji potencije nisu ucili) razumjeli, ponovimo sto su to potencije broja 10. Prisjetimo se:

100 = 1101 = 10102 = 100103 = 1000104 = 10 000

...10n = 1 0 . . . 0

︸ ︷︷ ︸

n nula

Eto, ponovili smo ono sto nam je ovdje potrebno znati o potencijama broja 10. Buduci da cemo se bavitibrojevima cije su znamenke jednake, primjerice brojem 8888, vazno je podsjetiti se sto znaci zapis broja. Zatopogledajmo sto se krije iza zapisa 3456. On predocava broj

3 · 103 + 4 · 102 + 5 · 10 + 6.

Dakle, svaki broj zapisan znamenkama a1, a2, . . . , an ima vrijednost:

a1a2 . . . an = a1 · 10n−1 + a2 · 10n−2 + · · · + an−1 · 101 + an.

Sada pogledajmo kako su brojevi zapisani jednakim znamenkama povezani s potencijama broja 10.

Primjer 1. Broj 99...9, zapisan pomocu 100 devetki, zapisimo krace sluzeci se potencijama.

Rjesenje. Primijetimo da se broj 9 moze zapisati kao 9 = 10 − 1, zatim da je 99 = 100 − 1 = 102 − 1,999 = 1000 − 1 = 103 − 1, itd. Stoga zakljucujemo da je nas broj jednak broju 10100 − 1. X

Evo kako cemo cinjenicu iz gornjeg primjera iskoristiti i za druge brojeve.

Primjer 2. Kako pomocu potencija zapisati broj koji ima k jednakih znamenaka?

Rjesenje. Zadani broj nazovimo A. Tada je A = aa . . . aa (gdje je a jedan od brojeva 1, 2, 3, . . . , 8, 9 i a jenapisan k puta). Postupak cemo provoditi na primjeru broja 8888 (dakle, za a = 8 i k = 4) i u opcem slucaju:

Najprije podijelimo broj sa znamenkom koja se ponavlja. Dobit cemo da je

8888

8= 1111,

A

a= 11 . . . 11.

Potom cijeli izraz pomnozimo s 9:

1Napomena urednistva. Uvod clanka je pisan za ucenike osnovnih skola koji potencije uce tek u 8. razredu. Ostale ideje iznesene uclanku koristit ce i srednjoskolcima.

http://playmath.petagimnazija.hr 15

Page 16: PlayMath 16-17

πlαy√

matχ9 · 8888

8= 9 · 1111 = 9999, 9

A

a= 99 . . . 9,

sto je:

104 − 1, 10k − 1.

Dakle, iz

9 · 8888

8= 104 − 1, 9

A

a= 10k − 1,

slijedi da je:

8888 = 8104 − 1

9, aa . . . a

︸ ︷︷ ︸

k

= a10k − 1

9.

X

Dobili smo lijepu dosjetku koja se moze cesto primijeniti. Evo nekoliko primjera takve primjene:

Primjer 3. Broj 22 . . . 2︸ ︷︷ ︸

l

77 . . . 7︸ ︷︷ ︸

k

zapisimo jednostavnije.

Rjesenje. Primijenimo ono sto znamo o zapisu broja i potencijama broja 10, pa broj zapisimo kao

2 · 10k+l−1 + 2 · 10k+l−2 + · · · + 2 · 10k + 7 · 10k−1 + · · · + 7 · 100 =

10k · (222 . . . 2︸ ︷︷ ︸

l

) + 77 . . . 7︸ ︷︷ ︸

k

= 10k · 2 · 10l − 1

9+ 7

10k − 1

9.

X

Primjer 4. Ako broj 10n pri dijeljenju sa 7 daje ostatak 2, dokazimo da broj koji ima 2n jednakih zna-menaka razlicitih od 7, nije djeljiv sa 7. (Uputa: Iskoristit cemo da je A2 − B2 = (A − B)(A + B)2.)

Rjesenje. Neka je X = xx . . . x. Koristeci nasu ”dosjetku” mozemo napisati da je X = x 102n−1

9. Kako je

102n = (10n)2, to primjenjujuci formulu za razliku kvadrata slijedi da je

X = x(10n − 1)(10n + 1)

9.

Kako ni x, ni 10n − 1, ni 10n + 1 nisu djeljivi sa 7 (zasto?), tako ni nas broj X nije djeljiv sa 7. X

Na ovaj se nacin dokazuju mnogi (bez nase dosjetke) za Matkace teski zadatci. Jedan takav zadatak (prviu kojemu sam se, pripremajuci se za natjecanje, osobno susreo s ovom dosjetkom) bio je ne tako davne 1988.godine na najvisem rangu natjecanja iz matematike za 8. razred. Evo tog zadatka.

Primjer 5. Dokazimo da razlika broja sastavljenog od 100 jedinica i broja sastavljenog od 50 dvojkipredstavlja kvadrat cijelog broja.

Rjesenje. Zapisimo razliku koristeci dosjetku:

10100 − 1

9− 2

1050 − 1

9=

10100 − 2 · 1050 + 1

9=

(1050 − 1

3

)2

.

X

2Napisani izraz naziva se formulom za razliku kvadrata.

16 PlayMath br. 16-17 (2008.)

Page 17: PlayMath 16-17

πlαy√

matχEvo sada nekoliko zadataka za samostalan rad.

ZADATAK 1. 3 Unutar znamenaka 4 i 9 broja 49 umetnuto je nekoliko cetvorki i isto toliko osmica. Dokaziteda je taj broj potpuni kvadrat.

ZADATAK 2. Dokazite da je11 . . . 1︸ ︷︷ ︸

2000

− 22 . . . 2︸ ︷︷ ︸

1000

= (33 . . . 3︸ ︷︷ ︸

1000

)2.

ZADATAK 3. Odredite zbroj6 + 66 + 666 + · · · + 66 . . . 66

︸ ︷︷ ︸

n

.

ZADATAK 4. Dokazite da je broj 777 . . . 7︸ ︷︷ ︸

l

djeljiv s 13 za neki l ∈ N ako i samo ako4 10l pri dijeljenju s 13 daje

ostatak 1.

UPUTE I RJESENJA NA STRANICI 70.

O autoru

Tvrtko Tadic dobitnik je dvije pohvale na Medunarodnim matematickim olimpijadama i brojnih nagrada nadrzavnim natjecanjima. Bio je dugogodisnji urednik PlayMath-a (glavni urednik od 2004. do 2007.). Autorje knjige Pripreme za matematicka natjecanja : za 4. razred gimnazije. Bio je clan Skupstine Hrvatskogmatematickog drustva od 2005. do 2008., a od 2004. do 2006. bio je i clan Izvrsnog odbora HMD-a.

2008. zavrsio je dodiplomski studij matematike.

Abstract

This article deals with the simple idea of writing many repeating digits in an integer with powers:

a

9(10n − 1) = aaa . . . a

︸ ︷︷ ︸

n digits

.

Later we show how this idea can be used to prove interesting properties of numbers comprised of many identicaldigits. The article was originally published in magazine Matka 44 (2003).

About the author

Tvrtko Tadic won two HONOURABLE MENTIONS at the International Mathematical Olympiads and manyprizes at the National Mathematics Competitions. He is an author of the book Pripreme za matematicka natje-canja : za 4. razred gimnazije (Preparation for Mathematical Competitions : for 12th grade). Tvrtko was aneditor of PlayMath (form 2004 to 2007 he was editor-in-chief of the magazine). He also served as a member ofthe Assembly and a member of the Executive Board of the Croatian Mathematical Society.

In 2008 Tvrtko graduated Mathematics at the Department of Mathematics University of Zagreb.

3Zadatak je 2000. godine bio zadan na opcinskom natjecanju iz matematike za prvi razred srednjih skola. U sluzbenom se rjesenjunije koristila ova dosjetka.

4Ako i samo ako (krace akko ili ⇔) je znak da treba dokazati da vrijede dva smjera. U ovom slucaju: (1) ako je izraz djeljiv s 13,onda vrijedi uvjet; i (2) ako vrijedi uvjet, onda je izraz djeljiv s 13.

http://playmath.petagimnazija.hr 17

Page 18: PlayMath 16-17

πlαy√

matχ

Igra sest stotica

Marko Horvat, Filip Niksic

Ovaj clanak objavljen je u Matki broj 44 (2003.).

U Matki br. 41. nalazi se vrlo zanimljiv problem u obliku mozgalice: u kolu na slici treba rotirati krugovedok zbroj brojeva u svakom stupcu ne bude jednak 100.

Slika 1.

Rjesenje: Pokusajmo zadatak rijesiti na sto jednostavniji nacin:

/* mozgalica "Sest stotica"

Ispisuje:

31 3 9 15 20 22

23 7 27 17 12 14

0 11 16 28 13 32

8 21 19 24 2 26

34 33 0 10 18 5

4 25 29 6 35 1

18 PlayMath br. 16-17 (2008.)

Page 19: PlayMath 16-17

πlαy√

matχ*/#include <stdio.h>

int polje[6][6] = {31,3,9,15,20,22,23,7,27,17,12,14,0,11,16,28,13,32,19,24,2,26,8,21,0,10,18,5,34,33,35,1,4,25,29,6};

int main() {

int p[6]={0,0,0,0,0,0},da,i,j,s;

do {da=1;for (i=0;i<6;i++) {

s=0;for(j=0;j<6; s+=polje[j][(p[j++]+i)%6]);if(s!=100) {

da=0;break;}

}if(da) {

for(i=0;i<6;i++) {for(j=0:j<6;printf("%4d",polje[i][(p[i]+j++)%6]));printf("\n");}

break;}

elsefor(i=5;i>=0;i--) {

if(++p[i]>5) p[i]=0;else break;}

}while(p[0]+p[1]+p[2]+p[3]+p[4]+p[5]);

return 0;}

Mozgalicu smo rijesili brute force algoritmom u programskom jeziku C. Program pretrazuje sve moguce kombinacijenase mozgalice i pronalazi rjesenje. Ovaj nacin rjesavanja problema nije osobito popularan buduci da se najcesce radi ovrlo sporim algoritmima, ali kako u nasem zadatku nema previse kombinacija (za pojmove racunala), ovakav je programi vise nego zadovoljavajuci.

Dvodimenzionalnu matricu, tj. polje dimenzija 6 × 6 ispunili smo vrijednostima sa slike tako da stupci matriceodgovaraju stupcima mozgalice. Vektor p sluzi nam da odredimo za koliko je mjesta odredeni red pomaknut udesno.Primjerice, ako elementima vektora odgovaraju vrijednosti [0; 0; 3; 0; 0; 1], znaci da je treci red pomaknut za3 mjesta, a sesti red za 1 mjesto udesno.

Glavni dio programa odvija se u do...while petlji. Prvo provjeravamo je li za trenutni pomak (koji cuvamo uvektoru p) zbroj u svim stupcima 100. Ako je navedeni uvjet ispunjen, znaci da smo pronasli rjesenje pa ga ispisujemo iizlazimo iz petlje (naredba break). U slucaju da uvjet nije ispunjen, tj. za trenutni pomak redova rjesenje nije postignuto,vektor p mijenjamo tako da element s najvecim indeksom uvecamo za 1 (sto znaci da najdonji red pomaknemo za jedno

http://playmath.petagimnazija.hr 19

Page 20: PlayMath 16-17

πlαy√

matχmjesto udesno). Ukoliko to rezultira vracanjem reda u pocetni polozaj, tada indeks prije njega uvecavamo za 1 (dakle,red iznad pomicemo za jedno mjesto udesno) i tako redom. Ukratko, povecavanjem elemenata vektora p simuliramo okretanje nase mozgalice.

Pokretanjem programa dobit cemo sljedeci ispis:

31 3 9 15 20 2223 7 27 17 12 140 11 16 28 13 328 21 19 242 2634 33 0 10 18 54 25 29 6 35 1

Zbrojite li stupce, vidjet ce te da je ovo tocno rjesenje, a pazljivijim promatranjem vidjet cete da je dobiveno okretanjemcetvrtog i petog reda za 2 mjesta udesno i sestog reda za 2 mjesta ulijevo.

Malom preinakom programa tako da se ne zaustavlja cim pronade prvo rjesenje moguce je provjeriti i je li ovo jedinorjesenje mozgalice. Ako tako prilagodimo program, on ce ispisati jos 5 rjesenja, no promatranjem tih rjesenja dolazimodo zakljucka da su sva medusobno jednaka. Dakle, dobiveno je rjesenje ujedno i jedino rjesenje ove mozgalice.

O autorima

Marko Horvat bio je odgovorni urednik PlayMath-a (do 2004. godine) i inicijator izlazenja casopisa. Trenutno je prizavrsetku studija matematike (smjer: Teorijska matematika) na PMF - Matematickom odjelu u Zagrebu.

Filip Niksic bio je dugogodisnji urednik PlayMath-a, od njegovog osnutka do 2007. godine (urednik web-izdanja od2004. godine). Trenutno zavrsava studij matematike (smjer: Racunarstvo) na PMF - Matematickom odjelu u Zagrebu.

Abstract

In this article the authors give an algorithmic solution to the following brain-teaser: The circles in the picture have to berotated so that in each row the sum of numbers equals 100. The article was published in magazine Matka no. 44 (2003).

About the authors

Marko Horvat was the founding editor of PlayMath and served as editor until 2004. He is a Senior student of Mathematicsat the University of Zagreb, where he will graduate soon.

Filip Niksic served as editor of PlayMath until 2007. (He was the editor of the web-edition.) At the moment Filip is aSenior student of Computer Science and Mathematics at the University of Zagreb, where he will graduate soon.

20 PlayMath br. 16-17 (2008.)

Page 21: PlayMath 16-17

πlαy√

matχ

http://playmath.petagimnazija.hr 21

Page 22: PlayMath 16-17

πlαy√

matχ

Srinivasa Ramanujan

Rudi Mrazovic

Clanak je preuzet iz casopisa Matka br. 39 (2002.).

Srinivasa Ramanujan bio je jedan od najvecih indij-skih matematicara. Dao je velik doprinos teoriji brojeva,a ponajvise je proucavao elipticke funkcije, verizne raz-lomke, te beskonacne redove. Roden je 22. listopada1887. u mjestu Erode, malom selu 400 km udaljenomod Madrasa. Uskoro se preselio u Kumbakonam, 160 kmblize Madrasu, jer mu je otac tamo radio.

Slika 1. Srinivasa Ramanujan

S pet godina upisao se u osnovnu skolu, a vec je s11 godina poceo pohadati srednju. Godine 1900. sam jepoceo proucavati aritmeticke i geometrijske redove. Takoje vec 1902. godine otkrio metodu rjesavanja jednadzbitreceg stupnja, a uskoro je pronasao i metodu za rjesavanjejednadzbi cetvrtog stupnja.

Sljedece godine probao je naci rjesenja jednadzbe petogstupnja, ne znajuci da neke takve jednadzbe ne mogu bitirijesene u radikalima1, te naravno, nije uspio. Medutim, toga nije obeshrabrilo pa se nastavio baviti matematikom.Godine 1903. u skolskoj je knjiznici medu hrpom sta-rih knjiga pronasao i jednu knjigu iz matematike. U tojknjizi, staroj gotovo 50 godina, bili su zapisani osnovnimatematicki poucci i formule. Ramanujan je bio zadivljenonime sto je procitao u toj knjizi, te se sve vise zanimao zamatematiku.

Godine 1904. istrazivao je zbroj beskonacnog reda∞∑

n=1

1

n, pa je izracunao broj e na cak 15 decimala2. Iste je

godine dobio jednogodisnju stipendiju za koledz, medutimsljedece godine stipendija mu nije obnovljena zbog togasto je Ramanujan radi matematike zanemarivao ostale pred-mete. Nasao se u novcanim poteskocama pa je zbog togapobjegao od kuce. Godine 1906. dosao je u Madras ne bili tamo pohadao koledz, a zatim bio primljen na tamosnjeSveuciliste.

Nastavio se baviti matematikom, a ovaj put ponajviseje proucavao geometrijske redove, te veze izmedu inte-grala i redova. Tek ce kasnije shvatiti da se bavio eliptickimfunkcijama. Na njegovu nesrecu, nakon tri mjeseca ucenjabolest mu je poremetila planove. Pao je na svim ispi-tima, osim na matematici. Sljedecih je godina nastavioproucavati matematiku, kao i inace, bez icije pomoci, ko-risteci se samo spomenutom knjigom. Sada se bavio veriznimrazlomcima i divergentnim redovima. Uskoro je poceoslati svoja otkrica tamosnjem matematickom listu. Nakonsto je objavio izvanredan clanak o Bernoullijevim broje-vima, napokon je dobio priznanje za svoj rad. U cijelomje Madrasu postao poznat kao matematicki genij.

Tako afirmiran ponovno je pokusao dobiti stipendiju,no opet nije uspio. Godine 1912. poslao je svoja otkrica odivergentnim redovima jednom engleskom profesoru, no

1Rjesenje algebarske jednadzbe zapisano pomocu konacnomnogo izvodenja operacija zbrajanja, oduzimanja, mnozenja, di-jeljenja i racunanja korijena s koeficijentima jednadzbe. Takojednadzbe oblika x5

− p2x − p = 0, gdje je p prost broj, nemogu biti rijesene u radikalima.

2Iracionalan broj koji oznacuje granicnu vrijednost niza„

1 +1

n

«

n

. Priblizna vrijednost e ≈ 2.718.

22 PlayMath br. 16-17 (2008.)

Page 23: PlayMath 16-17

πlαy√

matχ

Slika 2. G. H. Hardy

ovaj je iskreno priznao da ne moze razumjeti Ramanuja-nov rad. Svoje rezultate poslao je jos nekolicini profe-sora, ali oni se nisu udostojili ni odgovoriti. U sijecnju1913. godine Ramanujan salje svoja otkrica poznatim en-gleskim matematicarima Hardyju i Littlewoodu, nakonsto je vidio njihove knjige. Njih dvojica proucili su svepoucke koje je Ramanujan otkrio. Bili su zadivljeni. Na-ime, iako je medu Ramanujanovim pouccima bilo neko-liko vec poznatih, ostali su donosili odgovore na mnogetada nerijesene probleme.

Slika 3. Littlewood

Iako Ramanujan vecinu tih poucaka nije dokazao, Hardymu, odusevljen njegovim otkricima, 8. veljace 1913. go-dine salje odgovor, nakon cega Ramanujan uzvraca pi-smom i rijecima: ”Nasao sam prijatelja u vama ...” Rama-nujan, Hardy i Littlewood uskoro postaju veliki prijatelji.U svibnju 1913. Ramanujan je napokon uspio dobiti dvo-godisnju stipendiju. Vec sljedece godine Hardy ga pozivau Englesku, sto je Ramanujan odmah prihvatio. 17. ozujka1914. Ramanujan odlazi iz Indije, da bi za mjesec danadosao u Englesku gdje je s njim poceo raditi Littlewood.

Uskoro je i Littlewooda rat sprijecio da nastavi ucitiRamanujana, no njegovu ulogu odmah je prihvatio Hardy.Zajedno su proucavali elipticke integrale, geometrijske re-dove, funkcionalne jednadzbe, zeta funkciju i funkciju π(n)3.

Godine 1915. Ramanujana je pocelo muciti zdravlje.Lijecnici su mu preporucili da se vrati u Indiju, sto je on1919. godine i ucinio. Medutim, niti povratak u Indiju nijemu pomogao. Umro je sljedece godine. Pisma Hardyju iz1913. godine, te njegove biljeznice sadrzavale su gomilunedokazanih poucaka, a njihovim dokazivanjem razvile suse mnoge grane matematike. Prava vrijednost Ramanu-janovih otkrica vidjela se tek poslije njegove smrti. Kaoprimjer mozemo navesti da je veliki ruski matematicar J.M. Vinogradov ucinio ogroman korak u rjesavanju Gold-bachove hipoteze koristeci se upravo metodama Hardyja,Littlewooda i Ramanujana.

O autoru

Rudi Mrazovic dobitnik je tri bronacne medalje na Medunarodnimmatematickim olimpijadama i brojnih nagrada na drzavnimnatjecanjima. Bio je urednik casopisa PlayMath (od 2004.do 2006. pomocnik glavnog urednika). 2008. zavrsio jepreddiplomski studij Matematike i upisao diplomski studijMatematicke statistike na PMF - Matematickom odjelu.

3Funkcija koja daje broj prostih brojeva manjih ili jednakihn.

http://playmath.petagimnazija.hr 23

Page 24: PlayMath 16-17

πlαy√

matχAbstract

This article is about the Indian mathematician SrinivasaRamanujan. The article is from magazine Matka no. 39(2002).

About the author

Rudi Mrazovic won three bronze medals at the Internati-onal Mathematical Olympiads and many prizes at the Na-tional Mathematics Competitions. He was an editor ofPlayMath (assistant editor-in-chief from 2004 to 2006).In 2008 Rudi got a B.Sc. in Mathematics and is headingtowards a M.Sc. degree in Mathematical Statistics.

24 PlayMath br. 16-17 (2008.)

Page 25: PlayMath 16-17

πlαy√

matχ

Matematika pomocu racunala

U V. gimnaziji se, vec neko vrijeme, pokusava osuvremeniti nastava matematike. Tako se uz pomoc novih tehnologijapokusava ucenicima dati jedan novi uvid u matematiku i potaknuti ih na samostalno istrazivanje. U skoli su se odvijalibrojni programi kroz nastavu, a i izvan nje. Pritom su se koristili programi dinamicne geometrije (poput The Geometer’sSketchpad-a), programi za simbolicko racunanje (poput MAPLE-a) na osobnim racunalima i kalkulatori koji su imali tefunkcije.

U skoli je dugo godina djelovala grupa za Sketchpad i MAPLE i nekoliko programa Matematika uz pomoc racunala uorganizaciji Hrvatskog matematickog drustva.

U prilozima koji slijede bit ce predstavljeni neki od radova ucenika tokom ovih godina. Neki od njih su prikazivani usklopu raznih prezentacija, postavljeni na razne web-stranice (poput naseg web-izdanja i stranica distributera za Hrvatsku),a neki su objavljivani u casopisima i zbornicima radova.

Mathematics with the Computer Help

For several years, in in the 5th High School there have been several programs which tried to modernize mathematicslearning. Participating in these programs students learned to use dynamical geometry software (like The Geometer’sSketchpad) and computer algebra software (like MAPLE) on computers, and calculators which have similar functions.

In the following texts we will present some of the student projects made over the years. Some projects have beenpublished as the articles in some magazines or showed as presentations at workshops and at mathematician’s meetings.

http://playmath.petagimnazija.hr 25

Page 26: PlayMath 16-17

πlαy√

matχ

Raketa i avion

U animaciji raketa modeliran je napad raketom iz mora na avion. Animacija se moze preuzeti na stranicamawww.proven.hr (kao radovi ucenika gimnazije u datoteci matematika.gsp vidi stranicu RAKETA). Autor anima-cije je Hrvoje Torbasinovic ucenik 4. razreda (skolske godine 2003./2004.).

Slika 1.

Slika 2.

26 PlayMath br. 16-17 (2008.)

Page 27: PlayMath 16-17

πlαy√

matχ

Slika 3.

Slika 4.

http://playmath.petagimnazija.hr 27

Page 28: PlayMath 16-17

πlαy√

matχ

Slika 5.

Slika 6.

Abstract

The animation RAKETA made in The Geometer’s Sketchpad is modeling a misle attack on an airplane. This work canbe found on the web-page www.proven.hr.

28 PlayMath br. 16-17 (2008.)

Page 29: PlayMath 16-17

πlαy√

matχ

Suncani sat

U animaciji Suncani sat dana je animacija kretanja sunca i prikaz stanja na suncanom satu u zavisnosti o tomekretanju i mjesecu u godini. Ovu animaciju napravio je ucenik 2. razreda Ilija Cosic skolske godine 2005./2006.

Slika 1.

Slika 2.

http://playmath.petagimnazija.hr 29

Page 30: PlayMath 16-17

πlαy√

matχ

Slika 3.

Slika 4.

Abstract

The animation Suncani sat shows the shadow clock regarding the sun position and the time of the year.

30 PlayMath br. 16-17 (2008.)

Page 31: PlayMath 16-17

πlαy√

matχ

Dinamicni model Ptolomejevog sustava

U animaciji Ptolomejev sustav dana je animacija istoimenog sustava. Ovdje objavljujemo snimke. Animaciju jenapravio Hrvoje Torbasinovic skolske godine 2001./2002. (tada) ucenik 2. razreda, a moze se naci na PlayMath-ovomweb-izdanju u rubrici Racunala.

Slika 1.

Slika 2.

http://playmath.petagimnazija.hr 31

Page 32: PlayMath 16-17

πlαy√

matχ

Slika 3.

Slika 4.

32 PlayMath br. 16-17 (2008.)

Page 33: PlayMath 16-17

πlαy√

matχ

Slika 5.

Slika 6. Tragovi kretanja planeta i Sunca.

Abstract

The animation Ptolomej is showing the movement of the Earth, the Moon, the Sun and other planets in the Ptolomejsun system. The animation can be found in the Racunala section on our web edition.

http://playmath.petagimnazija.hr 33

Page 34: PlayMath 16-17

πlαy√

matχ

34 PlayMath br. 16-17 (2008.)

Page 35: PlayMath 16-17

πlαy√

matχ

O jednom preslikavanju

Tvrtko Tadic

Ovaj clanak nastao je u toku rada grupe za matematicki software skolske godine 2001./2002. Objavljen je u ZBOR-NIKU RADOVA 6. susreta nastavnika matematike RH odrzanom od 2.-5. srpnja 2002. godine u Zagrebu. Prenosimo gau cijelosti.

Prilikom obrade preslikavanja u prvom razredu (osne i centralne simetrije te inverzije) palo mi je na pamet – zasto nebih napravio neko preslikavanje po uzoru na simetrije koje smo tada radili, ali tako da bude ukljucena kruznica. Tako samdosao do sljedece ideje, koju sam uz ogromnu pomoc programa Geometer’s Sketchpad proveo u djelo.

Definicija 1. Preslikavanje P1

Neka je u ravnini zadana tocka A i kruznica k(S, r). Nacrtajmo polupravac iz tocke A kroz tocku S. Neka je tocka T onatocka presjeka polupravca i kruznice, koja ne pripada duzini AS. Tocku A preslikajmo centralnosimetricno s obzirom natocku T . Dobivena tocka A′ je slika tocke A po preslikavanju P1. (Vidi sliku 1.)

Slika 1. Preslikavanje P1

Primijetimo da se ovim postupkom mogu preslikati sve tocke ravnine osim sredista S odabrane kruznice k. Stoga jedomena ovog preslikavanja cijela ravnina osim tocke S. Isto tako mozemo primijetiti da je kodomena ovog preslikavanjacijela ravnina osim kruga i kruznice k.

Mnogi ce se zapitati zasto smo tocku T preko koje se vrsi preslikavanje odabrali bas na taj nacin i zasto je odabranpolupravac, a ne, recimo, pravac ili duzina? Polupravac smo izabrali zato sto duzina ne osigurava postojanje presjecista,a pravac bi uvijek dao dva presjecista pa ne bismo mogli odrediti kojem dati prednost. Tocka T je odabrana zato sto uslucaju kad se tocka A nalazi unutar kruznice, tada duzina AS nema zajednicku tocku s kruznicom, pa je T jedina tockadefinirana gornjim postupkom.

http://playmath.petagimnazija.hr 35

Page 36: PlayMath 16-17

πlαy√

matχDefinicija 2. Preslikavanje P2

Neka je A tocka ravnine i k(S, r) kruznica u toj ravnini. Nacrtajmo polupravac iz tocke S kroz tocku A. Tocku Apreslikajmo centralnosimetricno s obzirom na tocku H koja je sjecista polupravca i kruznice. Dobivena tocka A′ je slikatocke A po preslikavanju P2. (Vidi sliku 2.)

Slika 2. Preslikavanje P2

Primijetimo da je domena i ovog preslikavanja cijela ravnina osim tocke S. No kodomena je sada, za razliku odpreslikavanja P1, cijela ravnina (pa cak i tocka S!). Primijetimo, naime, da ako je tocka A izvan kruga tako da jer < |AS| < 3r, onda je A′ unutar kruga.

Kako su ta dva preslikavanja povezana? Postavimo sljedece pitanje: sto bi bilo kad bismo najprije primijenili jednopreslikavanje, a potom na rezultat tog preslikavanja primijenili drugo? Je li rezultat pocetna tocka? Jest, ako najprijeprimijenimo preslikavanje P1. Buduci da koodomena preslikavanja P2 nije podskup domene preslikavanja P1, to necemouvijek moci provesti opisano slozeno preslikavanje. Stoga je uvijek P2(P1(A)) = A, i vrijedi P1(P2(A)) = A iliP2(P2(A)) = A!

Kako se ova dva preslikavanja mogu opisati algebarski? Koristeci jednadzbu kruznice i pravca, te formulu za poloviste,dolazimo do sljedeceg:

Teorem 1. Za kruznicu k(S(0, 0), r) i tocku A(x, y) preslikavanje P1 je opisano formulom:

A(x, y) 7→ A′(

−2xr√

x2 + y2− x,

−2yr√

x2 + y2− y

)

.

Izvod ove formule je veoma jednostavan i mogao bi ga napraviti malo bolji osmas.

Teorem 2. Za kruznicu k(S(0, 0), r) i tocku A(x, y) preslikavanje P2 je opisano formulom:

A(x, y) 7→ A′(

2xr√

x2 + y2− x,

2yr√

x2 + y2− y

)

.

36 PlayMath br. 16-17 (2008.)

Page 37: PlayMath 16-17

πlαy√

matχPreslikavanje P1 pravca:

Na slici 3. pokazano je u sto preslikavanje P1 preslika pravac p. Mozemo vidjeti da je ovo preslikavanje znatnoslozenije od onih s kojima smo se upoznali u srednjoj i osnovnoj skoli.

Slika 3. Preslikavanje P1 pravca

Primijetimo da ako pravac prolazi tockom S, tada se pravac preslikava u samog sebe (osim tocke S za koju presli-kavanje nije definirano). Takoder primijetimo da je pravac p′ koji prolazi tockom S i koji je usporedan s pravcem p –asimptota dobivene krivulje. Naime, sto se tocka A na pravcu p vise udaljava od sredista S, to polupravac AS postaje”gotovo usporedan” s p, pa je slika A′ blizu usporednog pravca. Primijetimo takoder da nasa nova krivulja ima ”zvonolik”oblik i da nas podsjeca na Gaussovu krivulju.

Pokusajmo sada, koristeci teorem 2., dobiti jednadzbu ove krivulje. Odredivanje jednadzbe krivulje bilo bi tesko ikomplicirano preko teorema 1., buduci da je krivulja komplicirana, ovisi o polazaju pravca i ne mora niti biti funkcija.

Posljedica 1. Neka je pravac p zadan jednadzbom y = ax+ b i neka je kruznica k(S(0,0),r). Jednadzba krivulje P1(p) je:

2arx√

x2 + y2− ax − 2ry

x2 + y2+ y + b = 0.

Kako smo dosli do ovog rezultata? Mnogi ce misliti kako je on plod mukotrpnog rada i nekog ludog racuna. No nijetako. Bilo je dovoljno iskoristiti cinjenicu da je P2(P1(A)) = A, tj. P2(A

′) = A, gdje je A neka tocka pravca p. Dakle ako

tocka A’ ima koordinate (x, y), onda, koristeci teorem 2., slijedi da su koordinate od A

(

2xr√x2+y2

− x, 2yr√x2+y2

− y

)

.

Buduci da je tocka A na pravcu p, to za njezine koordinate (xA, yA) vrijedi: yA = axA + b, pa se se jednostavnimuvrstavanjem dobivenih koordinata tocke A dobije gornja jednadzba krivulje.

Preslikavanje P2 pravca

Ponovimo postupak iz prethodnog primjera primjenjujuci drugu definiciju. (Vidi sliku 4.)

Slika 4. Preslikavanje P2 pravca

Sto smo dobili? Odgovoriti na ovo pitanje mozda nam moze pomoci jednadzba te krivulje. Krecemo u njezinootkrivanje. No tu nastaje mali problem. Da bismo dobili njezinu jednadzbu, moramo koristiti prvo preslikavanje. Zatokrivulju moramo rastaviti.

http://playmath.petagimnazija.hr 37

Page 38: PlayMath 16-17

πlαy√

matχPrimijetimo da nastaje ”lom” krivulje u sredistu kruznice, jer preslikavanje P1 tu nije definirano te se tu nasa krivulja

dijeli na ”glavu” i ”tijelo”. ”Glava” (vidi sliku 5.) se natrag preslikava po drugom preslikavanju. Primijetimo takoder da”glava” nuzno ne mora postojati, ali zato ”tijelo” uvijek postoji!

Slika 5.

Posljedica 2. (POSLJEDICA O ”TIJELU” KRIVULJE) Neka je pravac p zadan jednadzbom y = ax + b i neka je kruznicak(S(0,0),r). Jednadzba ”tijela” krivulje P2(p) je:

2arx√

x2 + y2+ ax − 2y

x2 + y2− y − b = 0

Primijetimo da za svaku tocku A′ na ”tijelu” vrijedi P1(A′) = A, pa se jednadzba odredi slicno kao u posljedici 1.

Primijetimo da se moze izracunati i jednadzba ”glave”, no buduci da za svaku tocku A′ na toj krivulji, odnosno tomdijelu krivulje, vrijedi P2(A

′) = A, onda zapravo za ”glavu” vrijedi ista jednadzba kao i za prethodnu krivulju. To nasdovodi do zakljucka kako nase jednadzbe ne odreduju samo jednu krivulju nego vise njih. Kad bismo isti pravac preslikalii po prvom i po drugom preslikavanju, dobili bismo (ukoliko pravac ne bi bio udaljen za vise od 2r od sredista kruznice)da ”glava” i krivulja dobivena preslikavanjem P1 imaju istu jednadzbu. Zato je vazno naglasiti da taj dio ne mora postojatii da sve ovisi o polozaju pravca. To nas dovodi do zakljucka da nase krivulje zadovoljavaju jednadzbe, ali da te krivuljemozda nisu jedine koje tu jednadzbu zadovoljavaju.

38 PlayMath br. 16-17 (2008.)

Page 39: PlayMath 16-17

πlαy√

matχPreslikavanje P1 kruznice:

Ponovit cemo prethodni postupak i vidjet cemo sto je s kruznicom.

Slika 6.

Primijetimo da slika kruznice ne mora nuzno imati oblik kao na slici 6. Sve ovisi o medusobnom polozaju kruznica ki t.

Krenimo na izracunavanje jednadzbe te krivulje. Posluzit cemo se istim metodama kojima smo se sluzili i u teoremu3.

Posljedica 3. Neka je kruznica t(r1, S1(a, b)) i neka je kruznica k(S(0, 0), r). Jednadzba krivulje P1(t) je:

(

2rx√

x2 + y2− x − a

)2

+

(

2ry√

x2 + y2− y − b

)2

− r21 = 0.

I ovu smo posljedicu dobili kao i predhodne dvije (zbog primjene istog preslikavanja). Provjerimo sada tocnost ovetvrdnje u Geometer’s Sketchpadu.

Slika 7.

Kao sto vidimo na slici 7. (na kojoj slika kruznice ima drugaciji oblik nego na predhodnoj slici) mozemo vidjeti da jei ova jednadzba tocna.

Preslikavanje P2 kruznice:

http://playmath.petagimnazija.hr 39

Page 40: PlayMath 16-17

πlαy√

matχ

tk

Slika 8.

Primijetimo da se u ovom slucaju pojavljuje isto sto i prije. Oblik nije stalan, a i krivulja se mijenja, tj. ”lomi” usredistu kruznice preko koje se preslikava. Ponovo dolazi do kombinacija pri izracunavanju. Najprije cemo izracunatiprvu ”glavu”, tj. onu koja nema zajednickih tocaka sa spojnicom sredista dviju kruznica. Kao i u prethodnim primjerimadobijemo:

Posljedica 4. Neka je kruznica t(r1, S1(a, b)) i neka je kruznica k(S(0, 0), r). Jednadzba krivulje koja nema zajednickihdijelova sa spojnicom sredista P1(t) je:

(

2rx√

x2 + y2+ x + a

)2

+

(

2ry√

x2 + y2+ y + a

)2

− r21 = 0.

Primijetimo da, kao i kod drugog preslikavanja pravca, dio krivulje koji nismo racunali zadovoljava jednadzbu izposljedice 3. Isto tako, i u ovom slucaju drugi dio krivulje ne mora postojati.

Uporabimo Geometer’s Sketchpad i ispitajmo ispravnost zakljucka. (Slika 9.)

Dakle, nas je racun dobar.

40 PlayMath br. 16-17 (2008.)

Page 41: PlayMath 16-17

πlαy√

matχ

Slika 9.

Evo jos nekoliko slikovnih primjera za kraj:

Slika 10. Preslikavanje P1 cetverokuta

http://playmath.petagimnazija.hr 41

Page 42: PlayMath 16-17

πlαy√

matχ

Slika 11. Preslikavanje P2 elipse

Slika 12. Preslikavanje P2 trokuta

42 PlayMath br. 16-17 (2008.)

Page 43: PlayMath 16-17

πlαy√

matχ

Slika 13. Preslikavanje P1 parabole

Zakljucak

Vidimo da se mnoge cudne krivulje mogu napraviti ovim relativno jednostavnim preslikavanjima. Takoder vidimoi veliku korist programa Geometer’s Sketchpad, u kojemu su sve ove krivulje generirane i nacrtane. Program nam jekoristio i pri ispitivanju tocnosti rezultata. Vidjeli smo takoder da na mnoga pitanja nismo mogli dati odgovor. Nadamse da ce ovaj moj primjer potaknuti i druge da krenu u daljnja istrazivanja, kako nekih novih preslikavanja, tako i drugihgrana geometrije.

Abstract

This article is a student (research) paper presented at the 6th National Mathematics Teacher Meeting. The author explorestwo new transformations - reflection over the circle. Let k(S, r) be a circle and a point A be given in the plain. Let T bethe point of intersection of the circle k and the ray AS→ which is not on the segment AS. A′ is defined as the reflectionof point A to the point T . (See picture 1.) Similarly, a second transformation is defined (see picture 2). Equations ofcurves obtained by applying these transformations to lines and circles are given and checked with the software of dynamicgeometry - Sketchpad.

About the article

The article is a result of the author’s work in the Group for Mathematical Software in the 5th High School during theschool year 2001/2002.

http://playmath.petagimnazija.hr 43

Page 44: PlayMath 16-17

πlαy√

matχ

Republicko natjecanje 1960./1961.

Donosimo rezultate i zadatke 2. republickog natjecanja mladih matematicara. Na njemu je sudjelovalo 79 ucenika.

Rezultati

Poredak prve petorice.II. RAZRED

1. Prva nagrada nije dodijeljena.

2. Stojadinovic Zeljko, IX. gimnazija, Zagreb

3. Mioc Dubravka, II. gimnazija, Zagreb

4. Petris Nikica, VII. gimnazija, Zagreb

5. Solomun Zlatko, III. gimnazija, Zagreb

III. RAZRED

1. Volenec Vladimir, Gimnazija, Virovitica

2. Dodic Ivica, IX. gimnazija, Zagreb

3. Simat Marko, III. gimnazija, Zagreb

4. Vranic Ivo, III. gimnazija, Zagreb

5. Jerman Marijo, VII. gimnazija, Zagreb

IV. RAZRED

1. Marinovic Ante, VII. gimnazija, Zagreb

2.-4. Paar Vladimir, V. gimnazija, Zagreb

Andric Ivan, V. gimnazija, Zagreb

Matosin Spiro, V. gimnazija, Zagreb

5. Herkov Vesna, III. gimnazija, Zagreb

Zadatci

Na natjecanju su zadani sljedeci zadatci.

Drugi razred

1. Rijesi sustav jednadzbi

x + y + z = a − 1

x + 2y + 3z = 2a

x + 2y + 9z = 3a

i odredi a tako da rjesenja sustava budu nenegativna.

2. U jednadzbi 3x + 4y − 15 = 0 odredi x tako da razlika m − n odrezaka na koordinatnim osima bude jednaka 10.

44 PlayMath br. 16-17 (2008.)

Page 45: PlayMath 16-17

πlαy√

matχ3. Poznat je opseg raznostranicnog trokuta 2s. Ako nad najmanjom stranicom trokuta konstruiramo jednakostranican

trokut, njegov opseg je za 5 manji od opsega zadanog trokuta. Ali ako jednakostranican trokut konstruiramo nadnajvecom stranicom, njegov opseg je za 6 veci od opsega zadanog trokuta.

(a) Izrazi stranice promatranog trokuta kao funkciju njegova opsega 2s.

(b) Koje vrijednosti moze imati opseg 2s trokuta da trokut postoji? (Pokus za: 2s = 14, 2s = 8.)

4. Nacrtaj skup tocaka simetricnih zadanoj tocki A obzirom na tocke koje leze u zadanoj ravnini π udaljene d odtocke A. Tocka A ne pripada toj ravnini.

Treci razred

1. Izracunaj logaritamski7

5√

0.4 − 6√

2.5.

2. Dana je jednadzba x2 + ax + b = 0. Kako glasi jednadzba ciji su korijeni:

(a) za b veci;(b) b puta veci od korijena zadane jednadzbe.

3. Zadan kvadrat stranice a pretvori geometrijskom konstrukcijom u pravokutnik, kojemu bi opseg bio dva puta veciod opsega zadanog kvadrata.

4. Pravilna cetverostrana prizma presjecena je jednom ravninom koja prolazi njenim osnovnim bridom. Pod kojim jekutom nagnuta spomenuta ravnina prema bazi, ako je povrsina presjeka dva puta veca od povrsine baze?

5. Pozitivni brojevi x, y i z zadovoljavaju jednadzbu

log(x + y + z) = log x + log y + log z.

(a) Ako su x, y zadani nadi z.(b) Ako se zna da su x, y rjesenja jednadzbe u2 + au + b = 0 gdje su a, b pozitivni brojevi nadi z u funkciji od

a i b. Koji uvjet mora zadovoljavati a i b da odgovarajuca vrijednost za z bude pozitivna.

Cetvrti razred

1. Zbroj prvih n clanova jednog niza dan je izrazom Sn = 9.5n2 − 89.5n.

(a) Nadi opci clan tog niza i pokazi da je niz aritmeticki.(b) Pokazi da u promatranom nizu postoji jedan clan koji je dva puta veci od zbroja svojih prethodnika.

2. Zadana je jednadzba (4k + 1) sin2 x − (6k + 1) sin x + 3k − 6 = 0 gdje je k zadani realni broj.

(a) Za koje vrijednosti broja k su korijeni ove jednadzbe sinusi dvaju komplementarnih kutova?(b) Nadi sve kutove x koji zadovoljavaju jednadzbu u tom slucaju.

3. Dva su kuta trokuta odredena jednadzbama:

4tg x+tg y = 6,

16tg 2x−tg 2y = 8.

(a) Koliki je tangens treceg kuta?(b) Ako je zadan polumjer r opisane kruznice, nadi stranice trokuta.

4. Oko zadanog raznostranicnog trokuta ABC opisana je kruznica, a zatim povucen promjer DE paralelno sa strani-com AB. Krajnjim tockama tog promjera povucene su tetive DG i EF paralelno s ostalim dvjema stranicama.Dokazi da trokut ABC i cetverokut DEFG imaju jednaku povrsinu.

RJESENJA I UPUTE NA STRANICI 81.

http://playmath.petagimnazija.hr 45

Page 46: PlayMath 16-17

πlαy√

matχ

Republicko natjecanje 1964./1965.

Donosimo zadatke, rjesenja i rezultate 6. republickog natjecanja iz matematike.

O natjecanju

Sudjelovalo je ukupno 74 ucenika (31 ucenik 1. razreda, 18 ucenika 2. razreda, 5 ucenika 3. razreda i 20 ucenika 4.razreda), medu njima po prvi puta i ucenici 1. razreda srednje skole.

Natjecanje je odrzano 19. travnja 1965. godine u Zagrebu. Na natjecanje su pozvani ucenici koji su se najboljeplasirali na kvalifikacijskim natjecanjima koja su odrzana 19. ozujka 1965.

Na natjecanju je sudjelovalo 28 natjecatelja iz Zagreba; 12 iz Dakova; 6 iz Sinja; 4 iz Pule; po 3 natjecatelja iz Gospica,Osijeka, Splita i Vukovara; po dva natjecatelja iz Delnica, Siska, Varazdina i Virovitice; i po 1 natjecatelj iz SlavonskogBroda, Podravske Slatine, Vinkovaca, Zaboka i Zupanje.

Rezultati

Nagradeni su sljedeci ucenici.

I. razred

�������������������DRUGA NAGRADA

�����������������Branko Najman, V. gimnazija, ZagrebMarijan Turjak, XV. gimnazija, Zagreb�������������������

TRECA NAGRADA�����������������

Milan Zoric, V. gimnazija, Zagreb�������������������������������������������������������

II. razred

�������������������TRECA NAGRADA

�����������������Mladen Ladika, V. gimnazija, Zagreb�������������������������������������������������������

III. razred

�������������������TRECA NAGRADA

�����������������Dragan Milicic, VII. gimnazija, Zagreb�������������������������������������������������������

IV. razred

���������������������PRVA NAGRADA

�����������������Sime Ungar, V. gimnazija, Zagreb�������������������

DRUGA NAGRADA�����������������

Slobodan Bosanac, V. gimnazija, ZagrebMladen Grubic, V. gimnazija, Zagreb�������������������

TRECA NAGRADA�����������������

Pavle Senjanovic, VII. gimnazija, Zagreb�����������������������POHVALA

���������������������Predrag Cvitanovic, VII. gimnazija, Zagreb

Gordana Lukic, II. gimnazija, ZagrebVlado Perac, VIII. gimnazija, ZagrebEugenio Polla, Gimnazija ”b.S.”, Pula�������������������������������������������������������

46 PlayMath br. 16-17 (2008.)

Page 47: PlayMath 16-17

πlαy√

matχZadaci

Zadani su sljedeci zadaci.

I. razred

1. Izracunajte umnozak (x− a1)(x − a2)(x− a3) pa zatim rastavite na jednostavne faktore izraz I = (x− a− 2b +c)x2 − acx + 2b((a − c)x + ac) i skratite razlomak

I

2x2 − 2(2b− c)x − 4bc

nakon sto utvrdite za koje je vrijednosti od x taj razlomak definiran.

2. Razliku broja x i trecine njegove dvostruke vrijednosti uvecajte za 1. Koliko mora iznositi x da bidobiveni izraz

bio jednak3

2? Za koliko bi se promijenio x ako bismo zeljeli da vrijednost izraza bude triput veca.

3. Zadan je trokut ABC s tockom D na dtranici AB. Trokutu ABC treba upisati jednakostranicni trokut s vrhom uD. Diskusija.

4. Predocite graficki funkcije f1(x) = −x + 4 i f2(x) = 2x + 1. Izacunajte: a) povrsinu; b) stranice; c) jednutezisnicu trokuta sto ga odreduju grafovi ovih funkcija s osi x.

5. Neka su x i y dimenzije pravokutnika.

1) Napisite izraz za koji se uveca povrsina pravokutnika ako se x uveca za 3 m, a y za 4 m.

2) Neka je uvecanje pravokutnika ∆P = 36 m2. Pretpostavimo da se x i y mijenjaju uz konstantno ∆P .

a) Prikazite graficki kako tada y zavisi o x.

b) Izracunajte vrijednosti od x i y koje se odnose kao brojevi3

4i5

3.

II. razred

1. Razliku (√

2 − 1)x i polovice njegove trostruke vrijednosti umanjite za 5. Koliki mora biti x da bi dobiveni izraz

bio jednak −√

2

3+ 1? Rjesenje napisite tako nazivnik bude racionalan.

2. Prikazite graficki funkcije: a) (√

x − 1)2, b)√

(x − 1)2, c) |x − 1|, d) x − 1. Za koje vrijednosti od x vrijedi:1) |x − 1| =

(x − 1)2, 2)√

(x − 1)2 = (√

x − 1)2, 3)√

(x − 1)2 = x − 1, 4) (√

x − 1)2 = x − 1?

Rijesite jednadzbe: A) (√

x − 1)2 = 3, B)√

(x − 1)2 = 3, C) |x − 1| = 3, D) x − 1 = 3.

3. U kruznici je zadana tetiva. Konstruirajte toj tetivi paralel nu tetivu tako da je polumjeri koji pripadaju krajeimazadane tetive dijele na tri dijela.

4. Zadana je jednadzba 2x2 − (4a + 1)x + 3(2a − 1) = 0. Za koje vrijednosti parametra a ta jednadzba ima dvanejednaka realna rjesenja? Koje od dvaju rjesenja je vece?

5. Zadan je triedar V xyz1. Neka je C jedna tocka brida V z. Ortogonalna projekcija tocke C u ravnini xy neka je H .Ortogonalna projekcija tocke H na bridu V x neka je K, a ortogonalna projekcija iste tocke na na bridu V y nekaje L.

(i) Dokazite da su trokuti V CK i V CL pravokutni.

(ii) Stavite ∠(y, z) = a, ∠(z, x) = b, ∠(x, y) = c, a priklone kutove diedara kojima su bridovi x, y, z oznaciteredom s A, B, C.Dokazite da je |CH | = |CK| sin A, |CK| = |V C| sin b.

1Tri polupravca u prostoru x, y i z pocinju u vrhu V .

http://playmath.petagimnazija.hr 47

Page 48: PlayMath 16-17

πlαy√

matχ(iii) Dokazite:

sina

sin A=

sin b

sin B=

sin c

sinC.

(iv) Neka su α, β, γ prikloni kutovi bridova x, y, z prema nasuprotnim stranama triedra. Dokazite da jesin a sin α = sin b sin β = sin c sin γ.

III. razred

1. Nadite sve vrijednosti od x koje zadovoljavaju uvjet tg 2x < tg x. Razmotrite posebno slucaj tg x = −1.

2. Dokaziteπ2 < 10 ⇒ 1

log2 π+

1

log5 π> 2.

3. Vrhovi cetverokuta dijele kruznicu polumjera r na 4 dijela koji se odnose kao 1 : 2 : 3 : x. Najmanjoj stranicicetverokuta pripada sredisnji kut od 15◦.

1) Odredite x.

2) Koliki je odnos povrsine kruga prema povrsini cetverokuta?

3) Odredite povrsinu onog cetverokuta kojemu je najmanja stranica√

8.

4. Zadane su tocke A i B cija je udaljenost jednaka 2a. Odredite geometrijsko mjesto tocaka M za koje umnozak−−→MA · −−→MB konstantna vrijednost. Diskusija.

5. Odredite vrh V pramena pravaca y = (m − 1)x + 2m. U pramenu (V ) odredite pravce p1 i p2 tako da p1 prolazitockom A(−1, 3), a p2 tockom B(3,−1). Odredite kutove trokuta V AB. Kolika je udaljenost simetrale straniceAB od vrha V ?

IV. razred

1. Dokazite: Ako sin(y + z − x), sin(z + x − y) i sin(x + y − z) cine aritmeticki niz, onda i tg x, tg y i tg z cinearitmeticki niz. Kolika je suma drugog niza? Dokazite: Ako su x i z komplementni, da bi zadani nizovi biliaritmeticki mora vrijediti sin 2x tg y = 1 ili sin 2z tg y = 1.

2. Na osi x smjesteno je tijelo M . U trenutku t = 1 tijelo M se pocne gibati jednako promjenljivo. Ako je x apscisaod M , gibanje je dano funkcijom

x(t) = t2 − 4t + c.

Nacrtajte dijagram puta i dijagram brzine. Kolika je akcelercija?U trenutku t = 0 iz ishodista krene drugo tijelo M ′ koje se giba jednoliko po osi x brzinom v. Kako treba odabrativ da bi drugo tijelo stiglo prvo.

Neka je v =9

4. Odredite vrijeme i mjesto susreta M i M ′.

Kolika je srednja brzina prvog tijela izmedu dva susreta?

3. Zadan je polinom P (x) = 5x3 + ax2 + bx + c. Odredite konstantu k i parametre a, b, c tako da bude

P (x) + k(x − 1)P ′(x) + (x2 − 1)P ′′(x) = 0.

Ako dobivene vrijednosti uvrstimo u formulu za polinom P (x) dobit cemo polinom P0(x). Pokazite da je polinom

P0(x) djeljiv funkcijom (x − 1). Istrazite tok i nacrtajte graf funkcijeP0(x)

x − 1. Potrazite normalu na taj graf u tocki

u kojoj ona sijece os y. Kolika povrsina trokuta sto ga odreduje normala zajedno s koordinatnim osima?

4. Presijecite krivulje: f1(x) = ln x, f2(x) = ln x2, f3(x) = ln x3 pravcem x = m i polozite tangente u sjecistima.Pokazite da sve tri tangente prolaze istom tockom S. Koja je to tocka?Vrijedi li to opcenito za sve fr(x) = ln xr, ako je r proizvoljan realan broj? Specijalno ako je r = 0. Koje svojstvoima pramen krivulja fr(x) = ln xr .Koji je vrh tog pramena? Koje je geometrijsko mjesto tocka S?

48 PlayMath br. 16-17 (2008.)

Page 49: PlayMath 16-17

πlαy√

matχ5. Dokazite (a1 + a2)

5 je djeljivo s 5, ako je a51 + a5

2 djeljivo s 5. Poopcite to na proizvoljan broj pribrojnika i izpoopcenja izvedite da je zbroj petih potencija prirodnih brojeva djeljiv s 5, ako je zbroj baza djeljiv s 5.

Povjerenstvo za natjecanja

Ondasnja Republicka komisija za takmicenja u matematici ucenika skola II stupnja radila je skolske godine 1964./1965.u sastavu:PREDSJEDNIK:

Ignacije Smolec, Zavod za unapredivanje skolstva SR Hrvatske, ZagrebTAJNIK:

Jagoda Brkic, VII. gimnazija, ZagrebCLANOVI:

Stefanija Bakaric, XV. gimnazija, ZagrebVladimir Bencic, Pedagoska akademija, ZagrebNevenka Dravinac, XV. gimnazija, ZagrebJosefina Fuss, V. gimnazija, ZagrebDragutin Gustovic, Tehnicka skola, ZagrebDubravka Lemaic, Kemijska skola, ZagrebStjepan Lukic, Gradski zavod za skolstvo, ZagrebJosipa Luketic, Gimnazija ”Braca Ribar”, ZagrebAnda Matinjak, IV. gimnazija, ZagrebTatjana Motika, 2. ekonomska skola, ZagrebFranjo Nedela, IV. gimnazija, ZagrebFabijan Radonic, Zavod za unapredivanje strucnog obrazovanja SR Hrvatske, ZagrebZdenka Siftar, Gradevinska tehnicka skola, Zagreb

Abstract

The problems and results of the Republic Mathematics Competition in Croatia are given. The solutions can be foundon page

http://playmath.petagimnazija.hr 49

Page 50: PlayMath 16-17

πlαy√

matχ

Savezno natjecanje 1967./1968.

Natjecanje je odrzano 12. svibnja 1968. godine. Ukupno je sudjelovalo 63 natjecatelja 8od toga 19 iz Hrvatske). Sudioniciiz Hrvatske postigli su sljedece rezultate.

IV. razred

���������������������PRVA NAGRADA

�����������������Dva natjecatelja dobila su PRVU NAGRADU.�������������������

DRUGA NAGRADA�����������������

Nije dodijeljena DRUGA NAGRADA.�������������������TRECA NAGRADA

�����������������Branko Najman, V. gimnazija, Zagreb

Hrvoje Galic, V. gimnazija, ZagrebDragutin Svrtan, Gimnazija, Zabok

Jos jedan natjecatelj dobio je TRECU NAGRADU.�����������������������POHVALA

���������������������Dinko Plenkovic, V. gimnazija, Zagreb

Milorad Mrkobrada, XV. gimnazija, ZagrebZvonko Kolaric, XV. gimnazija, Zagreb

Vjeran Hari, V. gimnazija, ZagrebJos osam natjecatelja dobilo je POHVALU.�������������������������������������������������������

III. razred

���������������������PRVA NAGRADA

�����������������Jedan natjecatelj dobio je PRVU NAGRADU.�������������������

DRUGA NAGRADA�����������������

Jedan natjecatelj dobio je PRVU NAGRADU.�������������������TRECA NAGRADA

�����������������Neven Mattes, XV. gimnazija, Zagreb�����������������������

POHVALA���������������������

Radovan Sedmak, XV. gimnazija, ZagrebMiroslav Doresic, XV. gimnazija, Zagreb

Zvonko Cerin, V. gimnazija, ZagrebJos sest natjecatelja dobilo je POHVALU.�������������������������������������������������������

II. razred

���������������������PRVA NAGRADA

�����������������Dvoje natjecatelja dobilo je PRVU NAGRADU.�������������������

DRUGA NAGRADA�����������������

Nije dodijeljena DRUGA NAGRADA.�������������������TRECA NAGRADA

�����������������Slobodan Tepic, XV. gimnazija, Zagreb�����������������������

POHVALA���������������������

Damir Henc, XV. gimnazija, ZagrebVelimir Baradek, XV. gimnazija, Zagreb

Jos sest natjecatelja dobilo je POHVALU.�������������������������������������������������������

50 PlayMath br. 16-17 (2008.)

Page 51: PlayMath 16-17

πlαy√

matχDonosimo zadatke s ovog natjecanja.

II. razred

1. Nadi cijelobrojna rjesenja jednadzbe√

x − 1

5+

y − 1

5=

√5.

2. U ravnini dane su cetiri tocke A, B, C i D. Konstruiraj kruznicu u toj ravnini koja prolazi kroz tocke A i B takoda su tangente povucene iz tocaka C i D jednake duljine.

3. Sfera prolazi polovistima triju bocnih bridova trostrane piramide i dodiruje svaki osnovni brid u polovistu.

(a) Dokazi da je piramida pravilna.

(b) Izracunaj polumjer te sfere ako su osnovni brid i visina bocne strane jednake a.

4. Odrediti sve realne brojeve a za koje nijedna vrijednost x koja zadovoljava nejednakost

ax2 + (1 − a2)a − a > 0

nije po apsolutnoj vrijednosti veca od 2.

III. razred

1. Rijesiti sustav jednadzbi

xm = yn, loga

x

y=

loga x

loga y.

2. Odrediti u ravnini xOy skup tocaka cije koordinate (x, y) (0 ≤ x ≤ 2π) zadovoljavaju relaciju

1

2(√

1 + sin x −√

1 − sin x) ≤ y ≤ 1 +1

2(√

1 + sin 4x +√

1 − sin 4x).

3. Vrh pravog kuta nalazi se u ishodistu koordinatnog sustava, a njegovi kraci klize po paraboli y2 = 2px i sijekuparabolu u tockama X i Y .

(a) Nadi skup tocaka (geometrijsko mjesto) polovista duzine XY .

(b) Pokazi da svi pravci XY imaju jednu zajednicku tocku.

4. Dokazi identitet

(x + y + z)2 =1

2[(x − y)2 + (y − z)2 + (z − x)2] + 3(xy + yz + zx).

(a) Odrediti kakvi trebaju biti x, y, z da bi izraz xy+yz+zx imao uz konstantan zbroj x+y+z = s maksimalnuvrijednost.

(b) Na temelju dobivenog rjesenja rijesi sljedeci problem: Poznato je da je vrijednost dijamanta proporcionalnakvadratu njegove tezine. Ako se jedan dijamant podijeli na 3 komada tezina x, y, z, pokazi da je ukupnavrijednost uvijek manja od vrijednosti cijelog komada i da je ona najmanja kad se podijeli na 3 komadajednake tezine.

http://playmath.petagimnazija.hr 51

Page 52: PlayMath 16-17

πlαy√

matχIV. razred

1. Neka su p i q prosti brojevi, broj q3 − 1 djeljiv s p, a broj p − 1 djeljiv s q. Dokazi da je

p = 1 + q + q2.

2. U razredu ima 25 ucenika. Dokazi da se od njih ne moze formirati vise od 30 kosarkaskih ekipa po 5 igraca, akoneke dvije ekipe nemaju vise od jednog igraca koji je clan obiju ekipa.

3. U pravokutnom trokutu ABC (∠B = 90◦) dan je kut α kod vrha A i visina na hipotenuzu |BA1| = h1. Iz tockeA1 spustena je visina A1B1 na BC, iz tocke B1 visina B1A2 na AC, iz tocke A2 visina A2B2 na BC, itd. U svakitrokut ABA1, BA1B1, A1B1A2, B1A2B2 itd. upisana je kruznica.

(a) Izracunaj zbroj povrsina krugova pridruzenih tim kruznicama.

(b) Odredi α tako da taj zbroj bude maksimalan.

4. Dokazi identitet

(x + y + z)2 =1

2[(x − y)2 + (y − z)2 + (z − x)2] + 3(xy + yz + zx).

(a) Odrediti kakvi trebaju biti x, y, z da bi izraz xy+yz+zx imao uz konstantan zbroj x+y+z = s maksimalnuvrijednost.

(b) Na temelju dobivenog rjesenja rijesi sljedeci problem: Poznato je da je vrijednost dijamanta proporcionalnakvadratu njegove tezine. Ako se jedan dijamant podijeli na 3 komada tezina x, y, z, pokazi da je ukupnavrijednost uvijek manja od vrijednosti cijelog komada i da je ona najmanja kad se podijeli na 3 komadajednake tezine.

RJESENJA NA STRANICI 70.

Abstract

In this note we are remembering the results and the problems the Croatian students on the Federal Mathematical Compe-tition held in 1968. The solutions to the problems are given on page 70.

52 PlayMath br. 16-17 (2008.)

Page 53: PlayMath 16-17

πlαy√

matχ

Savezno natjecanje 1968./1969.

Savezno natjecanje ucenika srednjih skola odrzano je 12. V. 1969. Natjecatelji iz Hrvatske postigli su sljedece rezul-tate:

���������������������PRVA NAGRADA

�����������������Crnac Davor, 2. razred, V. gimnazija, ZagrebCerin Zvonko, 4. razred, V. gimnazija, Zagreb

Tepic Slobodan, 3. razred, Matematicka gimnazija1, ZagrebJos troje natjecatelja dobilo je PRVU NAGRADU.�������������������

DRUGA NAGRADA�����������������

Henc Damir, 3. razred, Matematicka gimnazija, ZagrebJos petero natjecatelja dobilo je DRUGU NAGRADU.�������������������

TRECA NAGRADA�����������������

Cukic Ljubomir, 4. razred, Matematicka gimnazija, ZagrebJos jedan natjecatelj dobio je TRECU NAGRADU.�����������������������

POHVALA���������������������

Sedmak Radovan, 4. razred, Matematicka gimnazija, ZagrebJos osam natjecatelja dobilo je POHVALU.�������������������������������������������������������

Na natjecanju su zadani sljedeci zadatci.

Drugi razred

1. Koja relacija neovisna od m postoji izmedu rjesenja jednadzbe

(x2 − 6x + 5) + m(x2 − 5x + 6) = 0?

2. Dokazi da je za svaki prirodan broj n bar jedan od brojeva

33n + 22n i 33n − 22n

djeljiv sa 35.

3. Dane su dvije kruznice sa sredistima u O i O′ i polumjerima R i R/2; kruznice se dodiruju iznutra u tocki A.

4. U trostranoj piramidi kutovi bocnih strana pri vrhu piramide su pravi. Dokazi da vrh piramide, teziste osnovice isrediste opisane sfere oko piramide pripadaju istom pravcu.

Treci razred

1. Trostrana piramida OABC ima bocne bridove |OA| = a, |OB| = b, |OC| = c, a ∠AOB = ∠BOC = ∠COA =90◦. Ako se sa α, β, γ oznace redom kutovi kod vrhova A, B, C osnovice ABC, dokazi da je

ctg α : ctg β : ctg γ = a2 : b2 : c2.

2. Nadi sva realna rjesenja sustava jednadzbi

x1 + x2 + . . . + xn = 1,

x21 + x2

2 + . . . + x2n = 1,

...xn

1 + xn2 + . . . + xn

n = 1.

1Danasnja XV. gimnazija.

http://playmath.petagimnazija.hr 53

Page 54: PlayMath 16-17

πlαy√

matχ3. Dana je pravilna cetverostrana prizma s osnovnim bridom 2a i visinom a(1 +

√3). Sfera prolazi kroz sva cetiri

vrha donje osnovice i dodiruje gornju osnovicu. Izracunaj onaj dio povsine plasta prizme koji je unutar sfere.

4. Neki kratkovidni mudrac koji vidi predmete na udaljenosti manjoj od jedan metar, sklopio je ovakvu okladu: Akose bilo gdje na udaljenosti od d metara od njega postavi neki predmet, onda ce on, pod uvjetom da mu se poslijesvakog ucinjenog koraka (koji iznosi jedam metar) kaze samo da li se on na taj nacin priblizio predmetu ili ne, ukonacno mnogo koraka naci predmet, i broj ucinjenih koraka biti ce sigurno manji od 3

2d + 7. (Predmet se smatrapronadenim ako mu se mudrac priblizi na udaljenost manju od jedan metar.) Dokazi da ce mudrac dobiti okladu.

Cetvrti razred

1. Neki kratkovidni mudrac koji vidi predmete na udaljenosti manjoj od jedan metar, sklopio je ovakvu okladu: Akose bilo gdje na udaljenosti od d metara od njega postavi neki predmet, onda ce on, pod uvjetom da mu se poslijesvakog ucinjenog koraka (koji iznosi jedam metar) kaze samo da li se on na taj nacin priblizio predmetu ili ne, ukonacno mnogo koraka naci predmet, i broj ucinjenih koraka biti ce sigurno manji od 3

2d + 7. (Predmet se smatrapronadenim ako mu se mudrac priblizi na udaljenost manju od jedan metar.) Dokazi da ce mudrac dobiti okladu.

2. Ako je p neparan prosti broj i a cijeli broj koji nije djeljiv sa p, onda je jedan i samo jedan od brojeva

A = a1+2+...+(p−1) + 1,

B = a1+2+...+(p−1) − 1

djeljiv sa p.

3. U zajednickom dijelu unutrasnjosti parabola

x2 = p2 + 2py i x2 = p2 − 2py

upisati elipsu maksimalne povrsine.

4. Nadi sva realna rjesenja sustava jednadzbi

x1 + x2 + . . . + xn = a,

x21 + x2

2 + . . . + x2n = a2,

...xn

1 + xn2 + . . . + xn

n = an.

RJESENJA NA STRANICI 75.

Abstract

In this note we are remembering the results and the problems the Croatian students on the Federal Mathematical Compe-tition held in 1969. The solutions to the problems are given on page 75.

54 PlayMath br. 16-17 (2008.)

Page 55: PlayMath 16-17

πlαy√

matχ

Olimpijci V. gimnazije iz matematike

U 2008. odvijaju se dvije vazne obljetnice za PlayMath: 70 godina V. gimnazije i 50 godina matematickih natje-canja u Hrvatskoj. U skladu s tim podsjetit cemo se nekih ljudi koji su vezani uz ove dvije obljetnice.

Prisjecamo se matematickih olimpijaca V. gimnazije. V. gimnazija je imala 18 olimpijaca iz matematike koji suosvojili 2 srebrne i 14 broncanih medalja, te 6 pohvala.

���������������������������������������������������������������������

1965. (7. MMO, Berlin, DR Njemacka)

Sime Ungar, 4. razred

���������������������������������������������������������������������

1968. (10. MMO, Moskva, SSSR)

Branko Najman, 4. razred - BRONCANA MEDALJA

���������������������������������������������������������������������

1969. (11. MMO, Bukurest, Rumunjska)

Zvonko Cerin, 4. razred

���������������������������������������������������������������������

Do 1990. mladi matematicari iz Hrvatske sudjelovali u sklopu ekipe Jugoslavije na MMO-u. Prvih godina je svakazemlja sudionica MMO-a slala po 8 ucenika na to natjecanje, kasnije je zbog povecanja broja zemalja koje sudjeluju naMMO broj clanova ekipe smanjen na 6. Od 1992. Hrvatska samostalno sudjeluje na MMO-u.

���������������������������������������������������������������������

1998. (39. MMO, Taipei, Tajvan)

Filip Simic, 2. razred - BRONCANA MEDALJA

���������������������������������������������������������������������

1999. (40. MMO, Bukurest, Rumunjska)

Tomislav Pejkovic, 3. razred - BRONCANA MEDALJA

���������������������������������������������������������������������

http://playmath.petagimnazija.hr 55

Page 56: PlayMath 16-17

πlαy√

matχ���������������������������������������������������������������������

2000. (41. MMO, Taejon, Juzna Koreja)

Anja Baresic, 4. razredTomislav Pejkovic, 4. razred - BRONCANA MEDALJAFilip Simic, 4. razred - BRONCANA MEDALJA

���������������������������������������������������������������������

2001. (42. MMO, Washington, SAD)

Marko Zivkovic, 2. razred

���������������������������������������������������������������������

2002. (43. MMO, Glasgow, Velika Britanija)

Tomislav Gracin, 4. razred

Napomena. Ucenik 3. razreda Marko Zivkovic je na drzavnom natjecanjuizabran u olimpijsku ekipu, ali zbog poklapanja termina s olimpijadomiz fizike nije sudjelovao na 43. MMO.

���������������������������������������������������������������������

2003. (44. MMO, Tokyo, Japan)

Rudi Mrazovic 2. razred - BRONCANA MEDALJATvrtko Tadic, 3. razred - POHVALAMarko Zivkovic 4. razred - BRONCANA MEDALJA

���������������������������������������������������������������������

2004. (45. MMO, Atena, Grcka)

Matija Basic, 4. razred - BRONCANA MEDALJAGoran Drazic, 2. razred - BRONCANA MEDALJANikola Grubisic, 3. razred - POHVALARudi Mrazovic, 3. razred - BRONCANA MEDALJATvrtko Tadic, 4. razred - POHVALA

���������������������������������������������������������������������

56 PlayMath br. 16-17 (2008.)

Page 57: PlayMath 16-17

πlαy√

matχ���������������������������������������������������������������������

2005. (46. MMO, Merida, Meksiko)

Goran Drazic, 3. razred - SREBRNA MEDALJANikola Grubisic, 4. razred - BRONCANA MEDALJARudi Mrazovic, 4. razred - BRONCANA MEDALJAKristina Skreb, 4. razred - POHVALA

���������������������������������������������������������������������

2006. (47. MMO, Ljubljana, Slovenija)

Nikola Adzaga, 3. razred - BRONCANA MEDALJAGoran Drazic, 4. razred - SREBRNA MEDALJAVedran Palajic, 4. razred

���������������������������������������������������������������������

2007. (48. MMO, Hanoi, Vijetnam)

Nikola Adzaga, 4. razred - POHVALAPetar Sirkovic, 4. razred - BRONCANA MEDALJASasa Stanko, 4. razred - POHVALA

���������������������������������������������������������������������

2008. (49. MMO, Madrid, Spanjolska)

Adrian Satja Kurdija, 2. razred - BRONCANA MEDALJAGoran Zuzic, 3. razred - BRONCANA MEDALJAInes Marusic, 4. razred���������������������������������������������������������������������

Petnaest olimpijaca upisalo je studij matematike, troje studij elektrotehnike i jedna olimpijka studij biologije. Jedanolimpijac paralelno s matematikom studira i fiziku.

Prvo troje olimpijaca postali su sveucilisni profesori matematike.

http://playmath.petagimnazija.hr 57

Page 58: PlayMath 16-17

πlαy√

matχ

IMO participants form the 5th High School

In this year we celebrate 70th anniversary of the 5th High School and 50 years of mathematical competitions inCroatia.

In this report we list participants of the International Mathematical Olympiad form the 5th High School. 18students from 5th High School participated at the IMO and won 2 silver medals and 14 bronze medals, and 6 honourblementions.

���������������������������������������������������������������������

1965 (7th IMO, Berlin, DR Germany)

Sime Ungar, 12th grade student

���������������������������������������������������������������������

1968 (10th IMO, Moscow, USSR)

Branko Najman, 12th grade student - BRONZE MEDAL

���������������������������������������������������������������������

1969 (11th IMO, Bucharest, Romania)

Zvonko Cerin, 12th grade student

���������������������������������������������������������������������

Until the year 1990 students from Croatia participated at IMO as a part of the IMO team representing Yugoslavia. Atfirst, each participating country at the IMO sent a team of 8 students to the competition, later the number was reduced to6 (because the number of the participating countries increased). Since 1992. Croatia has been participating at the IMO asan independent country.

���������������������������������������������������������������������

1998 (39th IMO, Taipei, Taivan)

Filip Simic, 10th grade student - BRONZE MEDAL

���������������������������������������������������������������������

1999 (40th IMO, Bucharest, Romania)

Tomislav Pejkovic, 11th grade student - BRONZE MEDAL

���������������������������������������������������������������������

58 PlayMath br. 16-17 (2008.)

Page 59: PlayMath 16-17

πlαy√

matχ���������������������������������������������������������������������

2000 (41st IMO, Taejon, South Korea)

Anja Baresic, 12th grade studentTomislav Pejkovic, 12th grade student - BRONZE MEDALFilip Simic, 12th grade student - BRONZE MEDAL

���������������������������������������������������������������������

2001 (42nd IMO, Washington, USA)

Marko Zivkovic, 10th grade student

���������������������������������������������������������������������

2002 (43rd IMO, Glasgow, United Kingdom)

Tomislav Gracin, 12th grade student

Remark. 11th grade student Marko Zivkovic was elected to the IMO team atthe National Mathematics Competition. He did not participate at the IMO,because he participated at International Physics Olympiad at the same time whenthe IMO was scheduled.

���������������������������������������������������������������������

2003 (44th IMO, Tokyo, Japan)

Rudi Mrazovic 10th grade student - BRONZE MEDALTvrtko Tadic, 11th grade student - HONOURABLE MENTIONMarko Zivkovic 12th grade student - BRONZE MEDAL

���������������������������������������������������������������������

2004. (45th IMO, Athens, Greece)

Matija Basic, 12th grade student - BRONZE MEDALGoran Drazic, 10th grade student - BRONZE MEDALNikola Grubisic, 11th grade student - HONOURBLE MENTIONRudi Mrazovic, 11th grade student - BRONZE MEDALTvrtko Tadic, 12th grade student - HONOURBLE MENTION

���������������������������������������������������������������������

http://playmath.petagimnazija.hr 59

Page 60: PlayMath 16-17

πlαy√

matχ���������������������������������������������������������������������

2005 (46th IMO, Merida, Mexico)Goran Drazic, 11th grade student - SILVER MEDALNikola Grubisic, 12th grade student - BRONZE MEDALRudi Mrazovic, 12th grade student - BRONZE MEDALKristina Skreb, 12th grade student - HONOURABLE MENTION

���������������������������������������������������������������������

2006 (47th IMO, Ljubljana, Slovenia)Nikola Adzaga, 11th grade student - BRONZE MEDALGoran Drazic, 12th grade student - SILVER MEDALVedran Palajic, 12th grade student

���������������������������������������������������������������������

2007 (48th MMO, Hanoi, Vietnam)Nikola Adzaga, 4. razred - POHVALAPetar Sirkovic, 12th grade student - BRONZE MEDALSasa Stanko, 12th grade student - HONOURBLE MENTION

���������������������������������������������������������������������

2008 (49th IMO, Madrid, Spain)Adrian Satja Kurdija, 10th grade student - BRONZE MEDALGoran Zuzic, 11th grade student - BRONZE MEDALInes Marusic, 12th grade student���������������������������������������������������������������������

Fifteen IMO participants studied mathematics at university, three electrotechnics and one biology. One IMO partici-pant is studying mathematics and physics.

First three IMO participants became university professors.

60 PlayMath br. 16-17 (2008.)

Page 61: PlayMath 16-17

πlαy√

matχ

Grupe za natjecanja u V. gimnaziji

U V. gimnaziji vec dulji niz godina djeluju grupe za matematicka natjecanja1. Voditelji grupa najcesce su nastavnici istudenti (bivsi uspjesni natjecatelji), a ponekad im se pridruze i ucenici visih razreda.

Na grupama se obicno prenosi iskustvo starijih na mlade, kroz razne teme koje se obraduju. Raspravlja se o zadacima,diskutiraju rjesenja. . .

Voditelji prate ucenike na natjecanjima i cesto im daju korisne savjete kako o natjecanjima, tako i o daljem skolovanju.

Nazalost, voditelji grupa ovaj posao rade volonterski. Neke institucije poput PMF – Matematickog odjela prepoz-nale su ovaj oblik rada, te se studentima voditeljima grupa prilikom upisa na neki od diplomskih studija uz uspjeh napreddiplomskom studiju, kao jedan od dodatnih kriterija boduju i mentorstva ucenicima.

Rezultati ovakvog oblika rada ocituju se na rezultatima natjecanja ostavrenim u proteklim godinama.

Math Circles in the 5th High School

In the 5th High School for a number of years students have had the opportunity to participate in Math Circles -groups organized for the students with interest in mathematics. Circles are usually headed by teachers and universitystudents (former successful competitors), and in some cases by higher class students. Circles usually prepare students formathematical competitions.

Different topics are lectured and discussed at the meetings. Some problems and their solutions are often viewed frommany perspectives.

The Circle leaders accompany students at competitions and give them valuable information on further education.

Most of the work done in Circles is on a voluntary basis. The Department of Mathematics at University of Zagreb hasdecided to give extra points to students who work as the high school student tutors for mathematical competitions in thesecondary degree admission process.

The results of the Circle approach are seen through the results of competitions in the past years.

1Slicne grupe djeluju i za ostale predmete.

http://playmath.petagimnazija.hr 61

Page 62: PlayMath 16-17

πlαy√

matχ

Odredi broj!

1. (8. MEDITERANSKO MATEMATICKO NATJECANJE 2005.)Profesor je rekao Petru produkt dvaju prirodnih brojeva, a Slavku zbroj tih dvaju brojeva. Nijedan od njih na pocetku nijeznao broj koji je bio poznat drugom djecaku.

Jedan od djecaka rekao je drugome: Nema nacina da odredis moj broj.

Na to drugi djecak odgovori: Varas se, tvoj broj je 136.

Koji je broj rekao profesor svakom djecaku? Obrazlozite odgovor!

Odabrali smo sljedece rjesenje ovog zadatka.

Rjesenje. Neka je prvi broj P , a drugi D. Pretpostavimo da djecak koji je dao prvu izjavu ima produkt brojeva.

Kad bi drugi djecak mogao zakljuciti da je prvi broj 136, tada bi on imao cetiri mogucnosti: 137, 70, 38, 25. Kako su ovibrojevi zbrojevi dvaju prirodnih brojeva kojih je produkt jednak 136, drugi djecak ne moze pogoditi broj prvog.

Ako je prvi broj zbroj onda se iz izjave prvog ucenika lako vidi da prvi broj nije prost broj uvecan za jedan, jer drugi ondaprecizno pogada broj prvog. Drugi broj je oblika D = x(136 − x), 1 ≤ x ≤ 68.

Za x = 1 je D = 135 = 5 · 33 iz po dva rastava:

(5 · 3) · (3 · 3); 5 · 3 + 3 · 3 = 24 = 23 + 1,

(5) · (3 · 3 · 3); 5 + 3 · 3 · 3 = 32 = 31 + 1,

(5 · 3 · 3) · (3); 5 · 3 · 3 + 3 = 48 = 47 + 1,

(5 · 3 · 3 · 3) · (1); 5 · 3 · 3 · 3 + 1 = 136 = 135 + 1,

je 135 jedno rjesenje za broj D.

Ako je x ≥ 2 imamo ove rastave:

(1) · (x(136 − x)); 1 + x(136 − x),

(x) · (x(136 − x)); x + 136− x = 136,

koji su ocito razliciti. Zato drugi ucenik ne bi mogao pogoditi prvi broj.

Dakle, prvi ucenik je dobio broj 136, a drugi 135. X

Goran Drazic, 3. razred, V. gimnazija

Abstract

In this note the solution of the student Goran Drazic to the problem of guessing the number from the 8th MediterraneanMathematical Competition is presented.

62 PlayMath br. 16-17 (2008.)

Page 63: PlayMath 16-17

πlαy√

matχ

Tri rjesenja jedne nejednakosti

3. (7. MEDITERANSKO MATEMATICKO NATJECANJE 2004.)Ako su a, b, c pozitivni brojevi takvi da je

1 = ab + bc + ca + 2abc,

dokazite nejednakost

2(a + b + c) + 1 ≥ 32abc.

Kada vrijedi znak jednakosti?

Natjecatelji su ponudili razna rjesenja. Objavljujemo odabrana.

Prvo rjesenje. Dijeljenjem dane jednakosti s abc dobivamo

1

abc=

1

a+

1

b+

1

c+ 2,

ili

4abc =4

1

a+

1

b+

1

b+

1

1/2

.

Primjenom A-H nejednakosti slijedi

4abc =4

1

a+

1

b+

1

b+

1

1/2

≤ a + b + c + 12

4,

tj.

32abc ≤ 2(a + b + c) + 1.

X

Marko Popovic, 2. razred, V. gimnazija, ZagrebMartina Fekete, 4. razred, III. gimnazija, Osijek

Drugo rjesenje. Primjenom A-H nejednakosti dobiva se

a + b + c

3≥

A−H 31

a+

1

b+

1

c

=3

1

abc− 2

=3abc

1 − 2abc,

a odavde

2(a + b + c) + 1 ≥ 18abc

1 − 2abc+ 1 =

1 + 16abc

1 − 2abc.

Pokazat cemo

1 + 16abc

1 − 2abc≥ 32abc.

Supstitucijom P = abc ova nejednakost prelazi u njoj ekvivalentne

http://playmath.petagimnazija.hr 63

Page 64: PlayMath 16-17

πlαy√

matχ

1 + 16P ≥ 32P (1− 2P ),

64P 2 − 16P + 1 ≥ 0,

(8P − 1)2 ≥ 0.

Kako je posljednja nejednakost istinita, takva je i polazna. X

Matija Basic, 4. razred, V. gimnazije

Trece rjesenje. Pretpostavimo da vrijedi nejednakost

2(a + b + c) + 1 ≥ 32abc,

Mnozenjem lijeve strane s 1 = ab + bc + ca + 2abc dobivamo ekvivalentnu nejednakost

(ab + bc + ca + 2abc)(2(a + b + c) + 1) ≥ 32abc.

Nakon mnozenja i sredivanja dobivamo

1

12(2a2b + 2ab2 + 2b2c + 2bc2 + 2c2a + 2ca2 + 4a2bc + 4ab2c + 4abc2 + ab + bc + ca)

≥ 2abc,

a ova je nejednakost istinita jer se primjenom A-G nejednakosti lijeva strana moze ocijeniti s

12√

212a12b12c12 = 2abc

X

Goran Drazic, 2. razred, V. gimnazija

Abstract

In this note three different student proofs of the inequality from the Mediterranean Mathematical Competition are presen-ted.

64 PlayMath br. 16-17 (2008.)

Page 65: PlayMath 16-17

πlαy√

matχ

Dinamicko preslagivanje trokuta

Na drzavnom natjecanju 2004. godine 4. razredima srednje skole zadan je sljedeci zadatak.2. Unutar trokuta ABC s duljinama stranica a, b, c i odgovarajucim kutovima α, β, γ postoje tocke P i Q takve davrijedi

∠BPC = ∠CPA = ∠APB = 120◦,

∠BQC = 60◦ + α, ∠CQA = 60◦ + β, ∠AQB = 60◦ + γ.

Dokazite da vrijedi jednakost

(|AP | + |BP | + |CP |)3 · |AQ| · |BQ| · |CQ| = (abc)2.

Zadatak je izrazito slabo rjesen, a ponudeno je samo jedno ispravno rjesenje koje se prvi put obajvljuje.

Rjesenje. Uocimo da su tocke P i Q jedinstveno odredene unutar trokuta ABC. Naime, mozemo konstrirati kruznelukove nad npr. stranicama AB i BC s odgovarajucim sredisnjim kutovima koji se izuzev tocke B sijeku u jos tocnojednoj tocki unutar trokuta. Ta tocka je tocka P , odnosno Q.

Slika 1. Tocka jedinstveno odredena kutovima ε i ρ

Nad stranicama trokuta ABC konstruirajmo jednakostranicne trokute BA1C i CB1A (kao na slici). Pokazimo da jetocka P sjeciste pravaca AA1 i BB1. Neka je T sjeciste pravaca AA1 i BB1.

Trokut AA1C dobiven je rotacijom trokuta B1BC za 60◦ oko tocke C. Odavde (ili iz sukladnosti trokuta B1BC iAA1C) se dobiva jednakost ∠ATB = 120◦. Takoder, zbog sukladnosti vrijedi

∠TB1A1 = ∠BB1A1 = ∠A1AC = ∠TAC,

to povlaci da je TCB1A tetivni cetverokut, te je

∠CTA = 180◦ − ∠AB1C = 120◦.

Na isti nacin se pokaze da je TBA1C tetivan i ∠BTC = 120◦. Zbog jedinstvenosti je T = P . Iz slicnih razloga je P napresjeku pravaca BB1 i CC1, odnosno P je zajednicka tocka pravaca AA1, BB1 i CC1.

http://playmath.petagimnazija.hr 65

Page 66: PlayMath 16-17

πlαy√

matχ

Slika 2.

Slika 3. Promatrani trokuti

Zbog tetivnosti trokuta PBA1C i jednakostranicnosti trokuta BA1C slijedi ∠A1PC = ∠A1BC = 60◦. Odaberimotocku P ′ na duzini AA1 takvu da je ∠PCP ′ = 60◦. Trokut PCP ′ je jednakostranican, a to povlaci da je |PP ′| = |CP |.Uocimo da je trokut CP ′A1 dobiven rotacijom trokuta CPB oko tocke C za 60◦. Stoga je |BP | = |P ′A1|. A sada je

|AA1| = |AP | + |PP ′| + |P ′A1| = |AP | + |CP | + |BP |.

66 PlayMath br. 16-17 (2008.)

Page 67: PlayMath 16-17

πlαy√

matχZbog vec spomenutih rotacija vrijedi |AA1| = |BB1|. Na isti nacin se dobije |BB1| = |CC1|, odnosno

|AA1| = |BB1| = |CC1|.

Promatrajmo trokute ACA1, BAB1 i CBC1. Uocimo da je ∠CBC1 = 60◦ + β, ∠ACA1 = 60◦ + γ i ∠BAB1 =60◦ + α. Konstruirajmo trokut A′B′Q′ slican trokutu B1BC takav da je |A′B′|/|B1B| = c. Uocimo da je |A′B′| =c(|AP |+ |BP |+ |CP |), |A′Q′| = bc i |B′Q′| = ca. Nad B′Q′ konstruirajmo trokut B′C ′Q′ slican trokutu BB1A takavda je |B′C ′|/|AB| = a. Uocimo da je ∠B′Q′C ′ = 60◦ + α, |Q′C ′| = ab i |B′C ′| = a(|AP | + |BP | + |CP |).

Lako se dokazuje da je A′Q′C ′ slican trokutu AC1C, te da vrijedi ∠C ′Q′A′ = 60◦ + β, |A′C ′| = b(|AP | +|BP |+ |CP |). Uocimo da su trokuti ABC i A′B′C ′ slicni! Zbog odredenosti tocke Q omjerima trokuta slijede slicnostiABQ ∼ A′B′Q′, BCQ ∼ B′C ′Q′ i CAQ ∼ C ′A′Q′. Iz slicnosti dobivamo

|AQ|c

=|AQ||AB| =

|A′Q′||A′B′| =

cb

c(|AP | + |BP | + |CP |) =b

|AP | + |BP | + |CP | ,

i na isti nacin|BQ|

a=

c

|AP | + |BP | + |CP | ,

|CQ|b

=a

|AP | + |BP | + |CP | .

Mnozenjem posljednje tri jednakosti slijedi trazena jednakost. X

Tvrtko Tadic, 4. razred, V. gimnazija

Slika 4. Konstruirani i pocetni trokut

Abstract

The problem and the solution presented here were originally presented at National Mathematics Competition in 2004.The problem was solved by only one student and this solution is presented in the text. The solution cuts the triangle inparts, enlarges them, and joins the parts together into a new triangle. Out of the similarity of the old and the new trianglethe problem is solved.

http://playmath.petagimnazija.hr 67

Page 68: PlayMath 16-17

πlαy√

matχ

Cetiri kruznice i njihova zajednicka tocka

2. (9. MEDITERANSKO MATEMATICKO NATJECANJE 2005.)Neka su k i k′ dvije koncentricne kruznice sa sredistem u tocki O, te odgovarajucim polumjerima R i R′ (R < R′). ZrakaOx sijece k u tocki A. Nasuprotna zraka Ox′ sijece k′ u tocki B. Treca zraka Ot (razlicita od prethodne dvije) sijece k uE i k′ u F.

Dokazite da kruznice (OAE) i (OBF ) te kruznica s promjerom EF i kruznica s promjerom AB prolaze istomtockom.

Odabrali smo sljedece rjesenje.

Rjesenje. Neka je T sjeciste kruznica k = k(OAE) i k′ = k′(OBF ) razlicito od O. Cetverokuti AOET i BOTF sutetivni. Neka je ϕ = ∠OAE = ∠OEA. Tada je ∠BOF = 2ϕ, pa je ∠OBF = ∠OFB = 90◦ − ϕ.

xx'

t

F

T

E

AO I

k

k'

B

P

Slika 1.

Stoga vrijedi:

∠ATB = ∠ATO + ∠OTB = ∠AEO + ∠OFB= ϕ + (90◦ − ϕ) = 90◦;

∠ETF = ∠OTF − ∠OTE = (180◦ − ∠OBF ) − ∠OAE= 180◦ − (90◦ − ϕ) − ϕ = 90◦.

Time smo pokazali da tocka T lezi na kruznicama promjera AB i EF . X

Kristina Skreb, 4. razred, V. gimnazija

Abstract

The solution shows that four circles have a common point. The problem was originally posed at the 9th MediterraneanMathematical Competition.

68 PlayMath br. 16-17 (2008.)

Page 69: PlayMath 16-17

πlαy√

matχ

Polinomi i prosti brojevi

4. (9. MEDITERANSKO MATEMATICKO NATJECANJE 2005.)Neka je A skup polinoma stupnja 3 s vodecim koeficijentom 1 koji imaju sljedece svojstvo: za svaki polinom f(x) izA postoje pozitivan prost broj p koji ne dijeli 2004 i prirodan broj q koji je relativno prost s p i s 2004, takvi da jef(p) = 2004 i f(q) = 0.

Dokazite da postoji beskonacni podskup B ⊆ A takav da su grafovi svih polinoma iz B identicni do na translaciju.

Odabrali smo sljedece rjesenje.

Rjesenje. Neka je P skup svih polinoma i B skup polinoma definiran sa

B = {f | f(x) = (x + 1 − q)2(x − q), p ∈ P, q ∈ N, (p, q) =

(p, 2004) = (q, 2004) = 1, p − q = 2004}.Uocimo da je

f(q) = (q + 1 − p)2(q − q) = 0,

f(p) = (p + 1 − p)2(p − q) = 2004.

Dakle, B ⊆ A.

Uocimo da su grafovi polinoma f(x) i f(x + c), c ∈ R identicni do na translaciju. Ako imamo parove (p1, q1) i (p2, q2)takve da je p1 − q1 = p2 − q2 = 2004, tada je p1 = p2 + r, q1 = q2 + r pa dobivamo

f(x) = (x + 1 − p1)2(x − q1),

f(x + r)2 = (x + 1 − p1 + r)2(x − q1 + r) = (x + 1− p2)2(x − q2).

Zato je dovoljno pokazati da postoji beskonacno mnogo prostih brojeva p i prirodnih brojeva q s ovim svojstvima:

(i) p − q = 2004,(ii) (p, q) = 1,

(iii) (p, 2004) = 1,(iv) (q, 2004) = 1.

Zbog p ∈ P, p > q, uvjeti (ii) – (iv) su zadovoljeni uvjetom (i). Kada bi bilo, npr. d = (p, 2004) > 1, onda bi bilo d | q,pa (p, q) ≥ d > 1, sto je u suprotnosti s pretpostavkom.

Dakle, ako je zadovoljen uvjet (i), zadovoljeni su i uvjeti (ii), (iii) i (iv).

Kako postoji beskonacno mnogo prostih brojeva p takvih da je p > 2004, vrijedi tvrdnja zadatka. X

Rudi Mrazovic, 4. razred, V. gimnazija

Abstract

This is a solution of the problem posed at the 9th Mediterranean Mathematical Competition. It solves the problemconcerning prime numbers and polynomials.

http://playmath.petagimnazija.hr 69

Page 70: PlayMath 16-17

πlαy√

matχ

Upute i rjesenja

Jednake znamenke = potencija

1. Iskoristite formulu (A + B)2 = A2 + 2AB + B2

i isprobajte umetnuti (tj. napravite pokus) dva, tribroja da vidite kakve cete brojeve dobiti.

2. Iskoristite dosjetku i pogledajte gdje su parni eks-ponenti! (Ovaj je zadatak zapravo isti kao primjer5.)

3. Nakon sto iskoristite dosjetku, svedite sve na za-jednicki nazivnik i zbrojite, a onda iskoristite do-sjetku drugi put (jer ce se u brojniku pojaviti brojoblika 66 . . .6).

4. (1) Izjednacite s 13k i gledajte sto mora biti djeljivos 13. (2) Uvrstite u izraz tu djeljivost.

Savezno natjecanje 1967./1968.

II. razred

1. Kako bi lijeva strana jednadzbe bila definirana, moravrijediti x ≥ 1/5 i y ≥ 1/5, odnosno kako su bro-jevi x i y cijeli vrijedi x ≥ 1 i y ≥ 1. Takoder mora

vrijediti x − 1

5≤ 5 i y − 1

5≤ 5, odnosno x ≤ 5 i

y ≤ 5. Potrazimo ona rjesenja jednadzbe za koje jex ≤ y. Oznacimo y = x+k. Tada trazimo rjesenjajednadzbe

x + k − 1

5=

√5 −

x − 1

5

za x ∈ {1, 2, 3, 4, 5} i k ∈ {0, . . . , 5 − x}. Ovajednadzba, nakon dvostrukog kvadriranja, postaje

x =k2 − 10k + 29

20.

Pri danim uvjetima ova jednadzba ima jedino rjesenje(x, k) = (1, 1).

Dakle, jedina rjesenja pocetne jednadzbe su (x, y) ∈{(1, 2), (2, 1)}. Lako se provjeri da ova rjesenja za-ista zadovoljavaju.

2. Da bi tangente iz tocaka na neku kruznicu bile jed-nake duljine nuzno je i dovoljno da su tocke C iD jednako udaljene od sredista te kruznice. Dakle,srediste trazene kruznice pripada simetrali duzine

CD, tj. nalazi se na presjeku simetrala duzina ABi CD.

Konstrukcija lako provede na temelju gornjih za-kljucaka. Zadatak ima vise rjesenja ako simetraleduzina AB i CD imaju (jednu ili vise) zajednickihtocaka i ako kruznica sa sredistem u nekoj od tihtocaka na kojoj leze tocke A i B ne sadrzi u svojojunutrasnjosti tocke C i D.

3. (a) Neka je SABC dana piramida (sa vrhom S),A1, B1, C1, M , N , P redom polovista duzina SA,SB, SC, BC , CA, AB. Nadalje neka je s danasfera i k i k1 redom njezini presjeci s ravninamaABC i B1MC1. Kako su pravci BC, CA i ABtangente na sferu s, one su i tangente kruznice k(redom u tockama M , N , P ). Zato je

|AB| = 2|AP | = 2|AN | = |AC| =

= 2|CN | = 2|CM | = |BC|,

tj. trokut ABC je jednakostranican. Nadalje, akoje O1 srediste kruga k1, onda su tocke B i B1 si-metricne tockama C i C1 u odnosu na pravac O1M(vidi sliku 1.) pa slijedi

|B1B| = |C1C|, tj. |SB| = |SC|.

Slicno se dobiva |SA| = |SB|. Time je dokazanoda je SABC pravilna piramida.

Slika 1.

70 PlayMath br. 16-17 (2008.)

Page 71: PlayMath 16-17

πlαy√

matχ(b) Neka je O srediste sfere, R njen polumjer, T i T1 tezista trokuta ABC i A1B1C1, h = |ST |, y = |OT |,a = |AB| = |SM | (vidi sliku 2.). Tada je

h2 = |SM |2 − |MT |2 = a2 −(

1

3

a√

3

2

)

=11

12a2.

Kako su trokuti OTP i OT1A1 pravokutni (s pravim kutovima u vrhovima T i T1), vrijedi

R2 = |OT |2 + |TP |2 = |OT1|2 + |A1T1|2,

y2 +

(

1

3

a√

3

2

)2

=

(h

2− y

)2

+

(

2

3

a√

3

2

)2

.

Dalje lako dobivamo h2 − y = y, tj. y =

h

4i R =

3

8a.

Slika 2.

4. Za dano a ≥ 0 uvijek postoje realne vrijednosti za x sa proizvoljno velikom apsolutnom vrijednoscu koje zadovo-ljavaju danu nejednadzbu.

Pretpostavimo zato da je a < 0. Tada je uvjet zadatka ekvivalentan uvjetu da jednadzba

ax2 + (1 − a2)x − a = 0

ima oba korijena po apsolutnoj vrijednosti manja ili jednaka 2. (Lako se provjeri da su korijeni ove jednadzbe

uvijek realni i razliciti.) Kako su ti korijeni x1 = a i x2 = −1

a, navedeni je uvjet ispunjen ako i samo ako je

−2 ≤ a ≤ −1

2.

III. razred

1. Sustav ima smisla ako i samo ako je m, n ∈ R i a > 0, a 6= 1. Pri tome nepoznanice trebaju zadovoljavati x, y > 0i y 6= 1.

Oznacimo sa x1 := loga x i y1 := loga y, dani sustav se moze zapisati u obliku

mx1 = ny1, x1 − y1 =x1

y1. (1)

http://playmath.petagimnazija.hr 71

Page 72: PlayMath 16-17

πlαy√

matχAko je m = 0, onda je i n = 0 (jer nije dozvoljeno y1 = 0). Tada se sustav svodi na jednadzbu x1 − y1 = x1

y1

rjesenja koje su parovi oblika(

y21

y1−1 , y1

)

za y1 6= 1, pa su rjesenja danog sustava u tom slucaju svi parovi oblika

(

yloga y

loga y−1 ,y)

, y > 0, y 6= 1, y 6= a.

Promatrajmo slucaj m 6= 0. Rjesavanjem sustav (1) dobivamo da za m = n nema rjesenja, a za m 6= m rjesenje jepar

(n2

m(n − m),

n

n − m

)

.

Tada je rjesenje danog sustava(

an2

m(n−m) , an

n−m

)

.

2. Dana relacija moze se zapisati u obliku

1

2

(∣∣∣sin

x

2+ cos

x

2

∣∣∣−∣∣∣sin

x

2− cos

x

2

∣∣∣

)

≤ y ≤ 1 +1

2(|sin 2x + cos 2x| + |sin 2x − cos 2x|) .

Lijeva strana ove dvostruke nejednakosti ima vrijednosti:

L =

sin x2 za x ∈ [0, π/2],

cos x2 za x ∈ 〈π/2, 3π/2],

− sin x2 za x ∈ 〈3π/2, 2π],

a desna:

L =

1 + cos 2x za x ∈ [0, π/8] ∪ 〈7π/8, 9π/8]∪ 〈15π/8, 2π],1 + sin 2x za x ∈ 〈π/8, 3π/8] ∪ 〈9π/8, 11π/8],1 − cos 2x za x ∈ 〈3π/8, 3π/8]∪ 〈11π/8, 13π/8],1 − sin 2x za x ∈ 〈5π/8, 7π/8]∪ 〈13π/8, 15π/8],

Skup tocaka ravnine xOy za koje je 0 ≤ x ≤ 2π i L ≤ y ≤ D prikazan je na slici 3.

Slika 3.

3. (a) Neka pravac OX ima jednadzbu y = tx, t > 0. Tada pravac OY ima jednadzbu y = − 1tx. Njihovi presjeci sa

danom parabolom (osim tocke O) su

X

(2p

t2,2p

t

)

, Y (2pt2, 2pt).

72 PlayMath br. 16-17 (2008.)

Page 73: PlayMath 16-17

πlαy√

matχZato poloviste Z(x, y) duzine XY ima koordinate

x = p

(

t2 +1

t2

)

, y = p

(

−t +1

t

)

.

Eliminacijom parametra t dobivamo jednadzbu trazenog geometrijskog mjesta tocaka

y2 = p(x − 2p).

Lako se provjeri da je svaka tocka ove parabole poloviste jedne od opisani duzina XY .

Slika 4.

(b) Pravac XY ima jednadzbu(

x − 2p

t2

)

:

(

t2 − 1

t2

)

=

(

y − 2p

t

)

:

(

−t − 1

t

)

.

Direktno se da tu jednadzbu, za proizvoljno t, zadovoljavaju koordinate tocke (2p, 0).

4. (a) Iz danog identiteta (koji se dokazuje direktnom provjerom) slijedi

xy + yz + zx =1

3

[

(x + y + z)2 − 1

2

((x − y)2 + (y − z)2 + (z − x)2

)]

≤ 1

3s2,

pri cemu se jednakost postize ako i samo ako je x = y = z = s/3. Dakle, trazena maksimalna vrijednost je s2/3koja se postize ako su x, y i z medusobno jednaki.

(b) Tvrdnja zadatka slijedi iz dvostruke nejednakosti:

(x + y + z)2 ≥ x2 + y2 + z2 (2)= (x + y + z)2 − 2(xy + yz + zx)

≥ s2 − 2

3s2 =

1

3s2, (3)

pri cemu se jednakost u (2) postize ako i samo ako su dva od brojeva x, y, z jednaki nuli, a u (3) ako i samo ako jex = y = z.

http://playmath.petagimnazija.hr 73

Page 74: PlayMath 16-17

πlαy√

matχIV. razred

1. Iz q|p − 1 slijedi q < p, pa kako je p prost, iz p|q3 − 1 = (q − 1)(q2 + q + 1) i p > q − 1, slijedi p|q2 + q + 1.Neka je q2 + q + 1 = kp i suprotno tvrdnji neka k > 1. Tada iz q|p− 1 dobivamo p − 1 = lq za neko l ∈ N, pa je

q2 + q + 1 = k(lq + 1) = klq + k.

Odakle slijedi q|k − 1, pa je k ≥ q + 1. No onda je

q2 + q + 1 ≥ (q + 1)(lq + 1) ≥ q2 + 2q + 1,

sto je nemoguce. Dakle, mora biti k = 1 i p = q2 + q + 1.

2. Neka se pod danim uvjetima moze formirati n ekipa. Pronadimo sve dvoclane podskupove skupa od 25 ucenika −njih ima

(25

2

)

= 300. Svaka od n ekipa sadrzi po(

5

2

)

= 10 takvih dvoclanih podskupova. Zbog uvjeta zadatka

mora biti 10n ≤ 300, tj. n ≤ 30.

3. (a) Lako se pokazuje da je polumjer upisane kruznice pravokutnog trokuta sa hipotenuzom c i siljastim kutom αjednak

c

2(sin α + cosα − 1).

Slika 5.

Opisani trokuti (vidi sliku 5.)

4ABA1, 4BA1B1, 4A1B1A2, 4B1A2B2, . . .

su pravokutni sa siljastim kutom α i hipotenuzama jednakim redom

h1

sin α, h1, h1 sin α, h1 sin2 α, . . .

Zato je trazeni zbroj povrsina njihovih upisanih krugova

S(α) =π

4h2

1(sin α + cosα − 1)

[1

sin2 α+ 1 + sin2 α + . . .

]

4h2

1

(sin α + cosα − 1

sin α cosα

)2

.

(b) Izraz S(α) ima na intervalu 〈0, π/2〉 maksimum za α = π4 koji iznosi πh2

1(3 − 2√

2).

4. Vidi 4. zadatak za III. razred.

74 PlayMath br. 16-17 (2008.)

Page 75: PlayMath 16-17

πlαy√

matχSavezno natjecanje 1968./1969.

Drugi razred

1. Ako je m 6= −1, onda je dana jednadzba ekvivalentna s jednadzbom

x2 − 5m + 6

m + 1x +

6m + 5

m + 1= 0

i ima rjesenja x1 i x2 za koje vrijede jednakosti

x1 + x2 =5m + 6

m + 1= 5 +

1

m + 1,

x1x2 =6m + 5

m + 1= 6 − 1

m + 1.

Zbrajanjem ovih jednakosti dobivamox1 + x2 + x1x2 = 11.

2. Ako je n = 2k + 1, gdje je k ∈ {0, 1, 2, . . .}, onda je

33n + 23n = 272k+1 + 82k+1 = (27 + 8)

2k∑

j=0

(−1)j272k−j8j ,

pa je broj 33n + 23n djeljiv sa 27 + 8 = 35.

Ako je n = 2k, gdje je k ∈ {1, 2, 3, . . .}, onda je

33n − 23n = 36k − 26k = 729k − 64k,

= (729− 64)

k−1∑

j=0

729k−1−j64j ,

= 35 · 19k−1∑

j=1

729k−1−j64j ,

pa je broj 33n − 23n djeljiv sa 35.

3. Oznacimo sa k i k1 redom dane kruznice sa sredistima O i O1. Pretpostavimo da kruznica k2 sa sredistem O2 ipolumjerom x dodiruje kruznice k i k1 i pravac OO1 redom u tockama A, B i C, kao na slici 6.

Oznacimo c := |OC|. Kako je

|OO1| = R/2 + x, |OO2| = R − x, |O2C| = x, |CO1| = R/2 + c

i kako su trokuti O2CO i O2CO1 pravokutni (s pravim kutom u vrhu C), iz Pitagorinog poucka slijedi

(R

2+ c

)2

+ x2 =

(R

2+ x

)2

,

c2 + x2 = (R − x)2,

tj. Rc + c2 = Rx, c2 = R2 − 2Rx. Eliminacijom c iz ovih jednakosti dobivamo

x =4

9R.

Uocimo da je |OO1| = 17R/18 i |OO2| = 5R/9.

http://playmath.petagimnazija.hr 75

Page 76: PlayMath 16-17

πlαy√

matχ

A

B

C

O2

O1O

Slika 6.

Konstrukcija. Konstruirajmo kruznicu k3 sa sredistem u O1 i polumjerom 17R/18 i kruznicu k4 sa sredistem u O ipolumjerom 5R/9. Neka je O′

2 presjek kruznica k3 i k4. Konstruirajmo kruznicu k′2 sa sredistem O′

2 i polumjerom4R/9. Tada je k′

2 trazena kruznica.

Dokaz. Kruznice k3 i k4 se sijeku jer brojevi 17R/18, 5R/9 i R/2 zadovoljavaju nuzan i dovoljan uvjet da mogubiti stranice trokuta. Kako je

(17

18R

)2

>

(5

9R

)2

+

(R

2

)2

,

to je O1O′2O tupokutan sa tupim kutom kod vrha O. Prema tome tocka O nalazi se izmedu tocke O1 i podnozja

C ′. visine trokuta O1O′2O iz vrha O′

2. Oznacimo c′ := |C ′O| i h := |C ′O′2|. Kako su trokuti O′

2C′O i O′

2C′O1

pravokutni sa pravim kutom u vrhu C ′, dobivamo

h2 =

(5

9R

)2

− (c′)2 =

(17

18R

)2

−(

R

2+ c′

)2

.

Nadalje, lako dobivamo c′ = R/3 i h = 4R/9, pa krug k′2 dodiruje pravac OO1. Iz jednakosti

|O1O′2| =

17

18R =

R

2+

4

9R

slijedi da kruznica k′2 dodiruje kruznicu k1. Kako je

|OO′2| =

(c′)2 + h2 =5

9R = R − 4

9R,

dobivamo da se i kruznice k′2 i k dodiruju.

Diskusija. Kruznice k1 i k′2 sijeku se u dvije tocke pa zadatak ima dva rjesenja.

4. Neka je ABDA1 dana piramida i neka su bridni kutovi kod vrha A pravi. Neka je dalje ABCDA1B1C1D1

paralelepiped (sa jednom stranom ABCD i bridovima BB1, CC1, DD1). (Vidi sliku 7.)Neka je o poloviste duzine AC1 i T teziste trokuta A1BD. Tada je O srediste opisane sfere oko paralelepipedaABCDA1B1C1D1 (pa prema tome i oko piramide ABDA1). Pravac AC1 presjek sljedecih triju ravnina

ABC1D1, ACC1A1, ADC1B1.

Kako poloviste X duzine BD pripada pravcu AC , tezisnica A1X trokuta A1BD pripada ravnini ACC1A1. Slicnose pokazuje da i ravnini ABC1D1 i ADC1B1 sadrze po jednu tezisnicu trokuta A1BD. Prema tome tocka Tpripada presjeku te tri ravnine, odnosno pravcu AO.

76 PlayMath br. 16-17 (2008.)

Page 77: PlayMath 16-17

πlαy√

matχ

T

X

O

B1

C1

D1

C

A B

D

A1

Slika 7.

Treci razred

1. Primjenjujuci Pitagorin poucak i poucak o kosinusima dobivamo

cosα =|AB|2 + |AC|2 − |BC|2

2|AB||AC| ,

=a2 + b2 + a2 + c2 − b2 − c2

2√

a2 + b2√

a2 + c2,

=a2

√a2 + b2

√a2 + c2

.

Dalje je

sinα =√

1 − cos2 α =

√a2b2 + b2c2 + c2a2

√a2 + b2

√a2 + c2

.

Analogno dobivamo

cosβ =b2

√b2 + c2

√b2 + a2

, sin β =

√a2b2 + b2c2 + c2a2

√b2 + c2

√b2 + a2

,

pa lako slijedictg α

ctg β=

cosα

cosβ· sin β

sin α=

a2

b2.

Slicno se dokazuju i ostali odnosi.

2. Ocito je da sustav ima sljedecih n rjesenja

(1, 0, 0, . . . , 0), (0, 1, 0, . . . , 0), . . . (0, 0, . . . , 1).

Dokazimo da nema drugih rjesenja.

Neka je n = 2. Tada iz x1 + x2 = x21 + x2

2 = 1 slijedi

2x1x2 = (x1 + x2)2 − x2

1 − x22 = 0,

pa je jedan od brojeva x1 i x2 jednak 0, a drugi je jednak 1.

Neka je n = 3. Tada izx1 + x2 + x3 = x2

1 + x22 + x2

3 = x31 + x3

2 + x33

i jednakostix2

1 + x22 + x2

3 = (x1 + x2 + x3)2 − 2(x1x2 + x2x3 + x3x1),

http://playmath.petagimnazija.hr 77

Page 78: PlayMath 16-17

πlαy√

matχx3

1 + x32 + x3

3 = (x1 + x2 + x3)(x21 + x2

2 + x23 − x1x2 − x2x3 − x3x1) + 3x1x2x3

slijedi x1x2 + x2x3 + x3x1 = 0 i x1x2x3 = 0. Prema tome, x1, x2, x3 su rjesenja jednadzbe

x3 − x2 = (x − 1)(x − 0)(x − 0) = 0,

pa je jedan od jednak 1, a ostali 0.

Slucaj n ≥ 4 mozemo dokazati na dva nacina.

Prvi nacin. Uocimo da vrijedi

n∑

k=1

x2k(xk − 1)2

=n∑

k=1

x4k − 2

n∑

k=1

x3k +

n∑

k=1

x2k

= 1 − 2 + 1 = 0.

Iz prethodne jednakosti slijedi xk ∈ {0, 1}. Cime smo pokazali tocnost nase tvrdnje.

Drugi nacin. Iz jednakostix2

1 + x22 + . . . + x2

n = x41 + x4

2 + . . . + x4n = 1

dobivamox2

1 ≤ 1, x22 ≤ 1, . . . , x2

n ≤ 1.

Ako je neki od brojeva x21, x2

2,. . . , x2n (npr. x1) jednak 1, onda je svaki od preostalih brojeva (x2, . . . , xn) jednak

0. Ako je x21 < 1, x2

2 < 1, . . . , x2n < 1, onda je

x21 + x2

2 + . . . + x2n > x4

1 + x42 + . . . + x4

n,

pa n-torka (x1, x2, . . . , xn) nije rjesenje danog sustava.

Slika 8.

78 PlayMath br. 16-17 (2008.)

Page 79: PlayMath 16-17

πlαy√

matχ3. Neka su A, B, C, D, A1, B1, C1, D1 vrhovi dane prizme, pri cemu je osnovica kvadrat ABCD stranice 2a,

a AA1, BB1, CC1, DD1 su bridovi te prizme duljina kojih je jednaka (1 +√

3)a. Neka je dalje S presjekdijagonala kvadrata ABCD, S1 presjek dijagonala kvadrata A1B1C1D1, O i R redom srediste i polumjer danesfere i x := |OS|. (Vidi sliku 8.)

Trokut OSB je pravokutni s pravim kutom u vrhu S. Zato je

|S1S| − |S1O| = x =√

|OB|2 − |BS|2,

pa dalje dobivamo((1 +

√3)a − R)2 = R2 − 2a2, R = a

√3, x = a.

Ako su A2, B2, C2, D2 redom tocke na bridovima AA1, BB1, CC1, DD1, takve da vrijedi |AA2| = |BB2| =|CC2| = |DD2| = 2a, onda je ABCDA2B2C2D2 kocka, a dana sfera je opisana oko te kocke.Dio povrsine strane ABB1A1 prizme koji se nalazi unutar kruga opisanog oko kvadrata ABB2A2 jednak je

(2a)2 +1

4[(a

√2)2 − (2a)2] =

a2π

2+ 3a2.

Dio povrsine prizme koji se nalazi unutar sfere jednak je

4

(a2π

2+ 3a2

)

+ 4a2 = 2a2(8 + π).

4. Uvedimo pravokutni koordinatni sustav tako da se mudrac nalazi u ishodistu. Neka se predmet nalazi u tocki(x0, y0). Bez smanjenja opcenitosti mozemo pretpostaviti x0 ≥ 0, y0 ≥ 0. Mudrac moze traziti predmet nasljedeci nacin:

1) Najprije sa najvise 4 koraka nade kvadrant u kojem je smjesten predmet (tocnije, kvadrant u kojem se predmetnalazi ili od koga je udaljen manje od 1/2). (Vidi sliku 9.)

2) Zatim se krece onom dijelu x-osi koji je granica pronadenog kvadranta, sve dok ne pocne se udaljavati odpredmeta (npr. x koraka) pa se onda vrati jedan korak unazad. Pri tome vrijedi x − 3

2 ≤ x0 ≤ x − 12 .

3) Zatim se u uocenom kvadrantu krece paralelno y-osi (y koraka) dok ne ugleda predmet, Pri tome vrijedi

y − 1 +

√3

2≤ y0 ≤ y + 1.

PSfrag replacements

√3

2

Slika 9.

http://playmath.petagimnazija.hr 79

Page 80: PlayMath 16-17

πlαy√

matχUkupan broj koraka je najvise A = 4 + x + 1 + y, pa dobivamo

A = 5 + x + y ≤ 5 + x0 +3

2+ y0 + 1 −

√3

2

= x0 + y0 + 7 +1 −

√3

2<

√2d + 7 <

3

2d + 7.

Koristili smo tvrdnju: ako za pozitivne brojeve a i b vrijedi a2 + b2 < d2 ia

b= tg ϕ gdje je 0 < ϕ < π

2 , onda je

a + b < d(cosϕ + sin ϕ) =√

2d cos(π

4− ϕ

)

<√

2d.

Cetvrti razred

1. Vidi rjesenje 4. zadatka za treci razred.

2. Uocimo da je A − B = 2 i AB = (ap)p−1 − 1. Kako broj A − B nije djeljiv sa p (jer je p prost broj veci od 2),nisu oba broja A i B djeljivi sa p. Broj AB je djeljiv sa p prema malom Fermatovom teoremu. Zato je tocno jedanod brojeva A i B djeljiv sa p.

3. Ako se elipsa nalazi unutar podrucja ogranicenog parabolama x2 = p2 + 2py i x2 = p2 − 2py, gdje je npr. p > 0(vidi sliku 10.), onda se translacijom elipse tako da joj se presjek osi poklopi sa ishodistem koordinatnog sustava,

Slika 10.

a zatim rotacijom oko ishodista velika os elipse dode na x-os, dobiva elipsa koja je unutar zadanog podrucja.(Dokazi!)

Jednadzba trazene elipse sa maksimalnom povrsinom je oblika

x2

λ2b2+

y2

b2= 1, (4)

gdje je 0 < b ≤ p/2. Elipsa (4) je cijela sadrzana u zadanom podrucju ako i samo ako za svako y ∈ 〈0, b] vrijedi

x2p − x2

e = p2 − 2py − (λ2b2 − λ2y2) ≥ 0, (5)

gdje su xp i xe apscise tocaka parabole i elipse koje imaju istu ordinatu. Pri tome, kod elipse koja ima maksimalnupovrsinu za neko y0 ∈ 〈0, b] vrijedi x2

p − x2e = 0. Ako je y0 = b, onda je y0 = b = p/2, a ordinata tjemena

parabolex = λ2y2 − 2py + p2 − λ2b2 (6)

nije manja od b.

80 PlayMath br. 16-17 (2008.)

Page 81: PlayMath 16-17

πlαy√

matχDakle,

yT =p

λ2≥ b =

p

2,

odakle slijedi λ2 ≤ 2. Maksimalna povrsina je P1 = πλb2 = πp2

2√

2.

Ako je 0 < y0 < b, onda su koordinate tjemena parabole (6)

y0 =p

λ2,

x0 = p2 − λ2b2 − p2

λ2= 0,

odakle dobivamo dvije mogucnosti za λ2:

λ21 =

p2

2b2(p −

p2 − 4b2), λ22 =

p2

2b2(p +

p2 − 4b2).

Kako je p/λ21 > b i p/λ2

2 < b u obzir dolazi samo vrijednost λ = λ2. Povrsina odgovarajuce elipse je P2 = πλb2,pa slijedi

P 22 = π2λ2b4 =

π2p2

2(p +

p2 − 4b2).

Maksimum funkcije ϕ(b2) = b2(p +√

p − 4b2) se dostize za b = 2p2/9. Tada je λ2 = 3 i P2 =2p2π

3√

3> P1.

Trazena elipsa jex2

2p2/3+

y2

2p2/9= 1.

4. Ako je a = 0, onda sustav ima tocno jedno rjesenje

x1 = x2 = . . . = xn = 0.

Ako je a 6= 0, onda mozemo uvesti nove nepoznanice yk := xk

aza k ∈ {1, 2, . . . , n}. Tada je (y1, . . . , yn)

rjesenje sustava zadanog u 2. zadatku za treci razred. (Vidi rjesenje tog zadatka.) Nakon cega slijedi da su rjesenja(pocetnog) sustava (x1, . . . , xn) iz skupa

{(a, 0, 0, . . . , 0), (0, a, 0, . . . , 0), . . . (0, 0, . . . , 0, a)}.

Republicko natjecanje 1960./1961.

II. razred

1. x = − 12a − 3; y = 2a + 3; z = − 1

2a − 1. Da bi sve tri vrijednosti bile negativne, moraju biti zadovoljeni uvjeti

−1

2a − 3 < 0, 2a + 3 < 0, −1

2a − 1 < 0.

Rjesenje ovog sustava je

−2 < a < −3

2.

2. Za x = 0 dobivamo 4y − 15 = 0, tj. y = 154 = n odsjecak na osi y. Za y = 0 dobiva se 3λx − 15 = 0, tj.

x = 5λ

= m odsjecak na osi x. Iz m − n = 10 slijedi

5

λ− 15

4= 10 ⇒ λ =

4

11.

http://playmath.petagimnazija.hr 81

Page 82: PlayMath 16-17

πlαy√

matχ3. a) Rjesavanjem sustava a + b + c = 2s, 3a = 2s − 5, 3c = 2s + 6 dobivamo:

a =2s − 5

3, b =

2s − 1

3, c =

2s + 6

3.

b) Da bi trokut postojao mora svaka stranica biti manja od zbroja drugih dviju a veca od njihove razlike. No akoje ovaj uvjet ispunjen za jednu stranicu, onda je ispunjen i za ostale dvije stranice. Dovoljno je dakle zahtijevati dabude npr. c − b < a < b + c (c − b, a ne b − c zato, jer je c > b).

Kako je a < b < c, desna je strana nejednadzbe ispunjena za svaku vrijednost od 2s, a lijeva je zadovoljena za2s > 12. Za 2s = 14 dobiva se a = 3, b = 4 1

3 , c = 6 23 , trokut postoji.

Slika 11.

Za 2s = 8 dobiva se a = 1, b = 2 13 , c = 4 2

3 , trokut ne postoji.

4. Neka je N ortogonalana projekcija tocke A na ravninu π, |AN | = m i T proizvoljna tocka na π za d udaljena odA.

Tada je |NT | =√

d2 − m2 = r. Skup svih tocka T je dakle kruznica K(N, r).

Tocka T ′ je simetricna tocki A s obzirom na T lezi na pravcu AT tako da je d = |TT ′| = |AT |. Isto tako tockaN ′ simetricna tocki A obzirom na na N lezi na pravcu AN , tako da je |NN ′| = |NA| = m.

Koristeci poucke o proporcionalnosti u pramenu zraka, iz |AT | : |AT ′| = 1 : 2 i |AN | : |AN ′| = 1 : 2 slijediNT ‖ N ′T ′, tj. ∠AN ′T ′ = ∠ANT = 90◦. Skup svih okomica N ′T ′ povucenih u N ′ na zraku AN ′ je ravninaπ; paralelna s π i udaljena od π za m. Trazene tocke T ′ leze dakle u π.

Iz proporcionalnosti |NT | : |N ′T ′| = |AT | : |AT ′| = 1 : 2 slijedi ndalje da je |N ′T ′| = 2|NT | = 2r =2√

d2 − m2 (konstanta za sve tocke T ′). Skup svih tocaka T ′ je dakle kruznica k′(N ′, 2r).

III. razred

1. Koristeci logaritamske tablice dobiva se x ≈ −0.85442.

2. a) Prema Vieteovim formulama dobivamo

x′1 + x′

2 = x1 + b + x2 + b = −a + 2b,

x′1x

′2 = (x1 + b)(x2 + b) = x1x2 + b(x1 + x2) + b2 = b(1 − a + b).

Nakon ispustanja crtica dobiva se jednadzba

x2 + (a − 2b)x + b(1 − a + b) = 0.

b) x′1 + x′

2 = bx1 + bx2 = b(x1 + x2) = −ab; x′1x

′2 = b2x1x2 = b3. Jednadzba glasi x2 + abx + b3 = 0.

82 PlayMath br. 16-17 (2008.)

Page 83: PlayMath 16-17

πlαy√

matχ3. Iz 2x + 2y = 8a ili y = 4a − x uvrstavanjem u xy = a2 dobivamo x(4a − x) = a2 ili x2 − 4ax + a2 = 0.

Rjesenja su x1,2 = 2a±√

3; y1,2 = 2a∓ a√

3. a√

3 mozemo shvatiti kao visinu jednakostranicnog trokuta komeje stranica 2a. Zbog toga stranicu DA kvadrata ABCD produzimo za a preko A do E i nacrtamo jednakostranicnitrokut DEF kome je stranica 2a i prema tome visina |AF | = a

√3. Produzimo li sada BA preko A za 2a do G,

tada je |GF | = 2a + a√

3.

Slika 12.

Ako stranicu AD kvadrata produzimo preko D za a do H i odmjerimo |HK| = |AF | = a√

3, onda je |AK| =2a − a

√3. Trazeni je pravokutnik GLFM . (Vidi sliku.)

4. Spomenuti presjek BCKI je paralelogram, jer je BC ‖ IK i BI ‖ CK (presjecnice dviju paralelnih ravninatrecom su paralelne). (Vidi sliku.)

Kako je BC ⊥ AB (stranice kvadrata) i BC ⊥ BF (stranice pravokutnika), BC je okomit na pobocku ABFE iprema tome je BC ⊥ BI . Presjek je dakle pravokutnik.

P = 2B; am = 2a2 ⇒ m = 2a.

Kako je AB ⊥ BC i IB ⊥ BC to je α = ∠ABI prikloni kut presjeka prema bazi. Kako je cosα = am

= a2a

= 12 ,

to je α = 60◦.

5. Pocetna jednakost postajelog(x + y + z) = log(xyz),

sto povlaci da je xyz = x + y + z.

a) Ako su x i y zadani onda je

z =x + y

xy − 1,

ako je xy 6= 1.

b) Iz Vieteovih formula je x + y = −a i xy = b, pa je

z =−a

b − 1,

za b 6= 1. Da bi z bio pozitivan mora biti b − 1 < 0, odnosno b < 1.

http://playmath.petagimnazija.hr 83

Page 84: PlayMath 16-17

πlαy√

matχ

Slika 13.

IV. razred

1. a) an = sn − sn−1 = (9.5n2 − 89.5n)− (9.5(n − 1)2 − 89.5(n− 1)) = 19n − 99.b) a1 = 19 · 1 − 99 = −80; d = an − an−1 = (19n − 99) − (19(n − 1) − 99) = 19.Trazi se clan za koji je an = 2sn−1, tj.

19n − 99 = 2 · n − 1

2[−180 + (n − 2)19].

Dobiva se n = 11 dok n =27

11ne zadovoljava. Clan sa navedenim svojstvom je 11. po redu i iznosi 110.

2. Zbog x2 = 90◦ − x1 je sin x2 = cosx1 i sin2 x1 + sin2 x2 = sin2 x1 + cos2 x1 = 1. Nadalje, vrijedi (sin2 x1 +sin2 x2) − 2 sinx1 sin x2 = 1 i prema Vieteovim formulama

6k − 1

4k + 1− 2 · 3k − 6

4k + 1= 1.

Nakon sredivanja dobivamo 2k2 − 11k − 6 = 0 s rjesenjima k1 = 6 i k2 = − 12 .

b) Za k = 6 jednadzba glasi 25 sin2 x−35 sin x+12 = 0 i ima rjesenja sin x1 = 45 i sin x2 = 3

5 kojima odgovarajusiljasti kutovi α = 53◦7′50′′ i β = 36◦52′11′′. Opca su rjesenja x1 = α + n1 · 360◦, x′

1 = 180◦ − α + n2 · 360◦,x2 = β + n3 · 360◦ i x′

2 = 180◦ − β + n4 · 360◦.

1◦ Da bi bilo x1+x2 = 90◦, mora biti α+β+(n1+n3)360◦ = 90◦, odnosno zbog α+β = 90◦, (n1+n3)360◦ =0◦, a to ce biti za n3 = −n1.

2◦ Da bi bilo x1+x′2 = 90◦, mora biti α−β+180◦+(n1+n3)360◦ = 90◦. Kako α−β nije izrazen cijelim brojem,

to jednadzba ne moze biti valjana za cjelobrojne vrijednosti od n1 i n2. Isto tako ne moze biti x′1 + x2 = 90◦.

3◦ Da bi x′1 + x′

2 = 90◦ moralo bi biti 360◦ − (α + β) + (n3 + n4)360◦ = 90◦, tj. n2 + n4 = −1

2. Ovo ne moze

biti jer su n2 i n4 cijeli brojevi.

84 PlayMath br. 16-17 (2008.)

Page 85: PlayMath 16-17

πlαy√

matχSvi kutovi koji zadovoljavaju jednadzbu nalaze se u skupu

{(α + n · 360◦, α − n · 360◦) : n ∈ Z}.

Za k = − 12 jednadzba ima kompleksna rjesenja.

3. a) Iz tg x = 1, tg y = 12 dobivamo

tg z = tg (180◦ − (x + y)) = −tg (x + y) =tg x + tg y

1 − tg xtg y= −3.

b) Kako su x, y, z kutovi trokuta a tg x > 0, tg y > 0 i tg z < 0, to su x i y siljasti kutovi a z tup.

Posto je sin x = tg x

±√

1+tg 2x=

√2

2 , analogno sin y = 1√5

, a sin z = 3√10

to je a = 2r sin x = r√

2, b = 2r sin y =

2r√5

, c = 2r sin z = 6r√10

.

4. Povucemo li polumjer do tocaka A, B, C, F i G raspadnu se i trokut i cetverokut svaki na ti jednakokracna trokuta.Uocimo li kutove s paralelnim kracima vrijedi

∠ODG = ∠OGD = ∠BAC = α,

∠OEF = ∠OFE = ∠ABC = β.

Oznacimo li sa ϕ = ∠OGF = ∠OFG, onda je 2α + 2ϕ + 2β = 360◦ (zbroj unutrasnjih kutova cetverokuta), tj.α + β + ϕ = 180◦. Dakle, ϕ = γ.

Slika 14.

Povrsina

P (DOG) =1

2r2 sin(180◦ − 2α) =

1

2r2 sin 2α,

a tolika je i povrsina P (BOC), jer je ∠BOC = 2α (sredisnji kut kojem pripada obodni kut α).

Isto je tako

P (EOF ) = P (AOC) =1

2r2 sin 2β i P (GOF ) = P (AOB) =

1

2r2 sin 2γ.

http://playmath.petagimnazija.hr 85

Page 86: PlayMath 16-17

πlαy√

matχRepublicko natjecanje 1964./1965.

I. razred

1. Kada izmnozimo i sredimo zadane izraze, dobivamo

x3 − (a + 2b − c)x2 + (2b(a − c) − ac)x + 2abc = I,

x3 − (a1 + a2 + a3)x2 + (a1a2 + a1a3 + a2a3)x − a1a2a3.

Usporedivanjem polinoma, dobijemo

a1 = a, a2 = 2b, a3 = −c,

pa izraz I postajeI = (x − a)(x − 2b)(x + c).

RazlomakI

2x2 − 2(2b − c) − 4bcse moze zapisati u obliku

(x − a)(x − 2b)(x + c)

2(x − 2b)(x + c)=

x − a

2,

a razlomak je definiran za x 6= 2b, x 6= −c.

2. Izraz je x − 2x

3+ 1. Iz

x − 2x

3+ 1 =

3

2

slijedi x = 32 . Iz

x − 2x

3+ 1 =

9

2

slijedi x = 212 . Dakle, x bi se povecao za 9.

Slika 15.

3. Zamislimo li da je zadatak rijesen, tada prilikom rotacije stranice AC oko D za −60◦ vrh F ce pasti u tocku E,koja je sjeciste rotirane stranice A′C ′ i stranice CB.Isto rjesenje se dobije, ako stranicu BC rotiramo za 60◦ (u tom slucaju sjeciste ce biti tocka F .)Iz ovoga slijedi konstrukcija. (Vidi sliku.)

4. Koordinate tocaka su

A

(

−1

2, 0

)

, B(4, 0), C(1, 3).

(a) Povrsina trokuta P = 6 34 .

(b) Duljine stranica su |AB| = 4 12 , |BC| = 3

√2, |AC| = 3

2

√5.

(c) Duljina tezisnice tc = 14

√153 = 3

4

√17.

86 PlayMath br. 16-17 (2008.)

Page 87: PlayMath 16-17

πlαy√

matχ

Slika 16.

5. 1. Povrsina pravokutnika se poveca za (x + 3)(y + 4) − xy = 4x + 3y + 12.2. 4x + 3y + 12 = 36, odnosno y = − 4

3x + 8.

x : y =3

4:

5

3, odakle je x =

9

20y.

Iz dvije dobivene jednakosti slijedi y = 5, a x = 94 .

II. razred

1. Iz

(√

2 − 1)x − 1

2· 3(

√2 − 1)x − 5 = −

2

3+ 1,

mnozenjem sa zajednickim nazivnikom i nakon sredivanja dobivamo jednakost

−3(√

2 − 1)x = 36− 2√

6,

a odatle nazivnikom i nakon sredivanja dobivamo jednakost −3(√

2 − 1)x = 36 − 2√

6, a odatle

x =2√

6 − 36

3(√

2 − 1),

a nakon racionaliziranja nazivnika

x =2

3(2√

3 − 18√

2 +√

6 − 18).

2. Grafovi funkcija su dani na slici.

PSfrag replacements

y = (√

x − 1)2

PSfrag replacements

y =√

(x − 1)2 = |x − 1|

http://playmath.petagimnazija.hr 87

Page 88: PlayMath 16-17

πlαy√

matχ

PSfrag replacements

y = x − 1

Slika 17.

1. |x − 1| =√

(x − 1)2 za sve x ∈ R.

2.√

(x − 1)2 = (√

x − 1)2 za x ≥ 1.

3.√

(x − 1)2 = x − 1 za x ≥ 1.4. (

√x − 1)2 = x − 1 za x ≥ 1.

(a) (√

x − 1)2 = x − 1,x ≥ 1,x − 1 = 3,

x = 4.(b)√

(x − 1)2 = 3,(x − 1)2 − 9 = 0,

(x − 4)(x + 2) = 0,x1 = 4, x2 = −2.(c) |x − 1| = 3,x − 1 = ±3,x1 = 4, x2 = −2.(d) x − 1 = 4, x = 3.

3. Tvrdnja se dokazuje preko teorema o proporcionalnosti u pramenu pravaca. (Vidi sliku 18.)

4. D = (4a + 1)2 − 24(2a− 1) > 0 ili nakon sredivanja D = (4a − 5)2 > 0 uz a 6= 54 .

Za3

2< 2a − 1, a >

5

4;3

2> 2a− 1 za a <

5

4.

5. 1. KH je ortogonalna projekcija duzine KC, a V X je okomito na KH , pa prema poucku o tri normale slijedi daje V X ⊥ CK, tj. kut ∠CKV je pravi kut i prema tome je je trokut CKV pravokutan. Analogan je zakljucak zatrokut CLV .

2. a) Iz trokuta CKH s pravim kutom kod vrha H slijedi |CH | = |CK| sin A.b) Iz trokuta CKV (kut ∠CKV je pravi) slijedi |CK| = |CV | sin b. (Vidi sliku 19.)

3. Redom dobivamo

sin a

sin A=

|CL||CV ||CH||CK|

=|CL||CK||CV ||CH | ,

sin b

sinB=

|CK||V C||CH||CL|

=|CL||CK||CV ||CM | .

To povlacisin a

sin A=

sin b

sin B. (7)

88 PlayMath br. 16-17 (2008.)

Page 89: PlayMath 16-17

πlαy√

matχ

Slika 18.

Slika 19.

Projicirajuci tocku K na stranu yx dobije se slicnim razmatranjem, da je

sin c

sinC=

sin b

sin B. (8)

Iz (7) i (8) slijedisina

sin A=

sin b

sin B=

sin c

sinC.

4. Iz trokuta V KK ′ slijedi sin α = |KK′||V K′| , a iz trokuta V CH slijedi

sin γ =|CH ||V C| , sin a sin α =

|CL||KK ′||CV ||V K| , i sin c sin γ =

|KQ||CH ||V K||V C| . (9)

Iz trokuta KQK ′ slijedi |KK ′| = |KQ| sinB, a iz trokuta CHL slijedi, a iz trokuta CHL slijedi

|CH | = |CL| sinB. (10)

http://playmath.petagimnazija.hr 89

Page 90: PlayMath 16-17

πlαy√

matχSupstituirajuci (10) u (9) slijedi

sin a sinα =|CL||KQ| sinB

|V C||V K| i sin c sin γ =|KQ||CL| sinB

|V K||V C| ,

tj.sin a sinα = sin c sin γ. (11)

Projicirajuci L na stranu xz istim razmatranjem doslo bi se do zakljucka da je

sin b sinβ = sin c sin γ. (12)

Iz (11) i (12) slijedisin a sin α = sin b sin β = sin c sin γ.

III. razred

1. Iz tg 2x = tg x slijedisin 2x

cos 2x− sin x

cosx< 0,

a odatlesin 2x cosx − cos 2x sin x

cos 2x cosx=

sin x

cos 2x cosx=

1

cos 2x· tg x < 0.

Neka je:a) 1

cos 2x< 0, tg x > 0, a odatle π

4 < x < π4 ;

b) 1cos 2x

> 0, tg x < 0, a odatle 3π4 < x < π.

Za tg x = −1, tg 2x < −1 imamo:a) π

2 < 2x < 3π4 ili π

4 < x < 3π8 ;

b) 3π2 < 2x < 7π

4 ili 3π4 < x < 7π

8 .

2. Iz π2 < 10 logaritmiranjem po bazi π dobivamo

2 < logπ 10 = logπ 2 + logπ 5 =1

log2 π+

1

log5 π.

3. Vrhovi cetverokuta dijele kruznicu polumjera r na cetiri jednaka dijela, koji se odnose kao 1 : 2 : 3 : x. Najmanjojstranici cetverokuta pripada sredisnji kut od 15◦. Ako najmanjem luku odgovara sredisnji kut od 15◦, tada sutrazeni sredisnji kutovi: 15◦, 30◦, 45◦ i 270◦.Primjenom trigonometrijskih formula za povrsinu trokuta dobiva se

P (OCD) =1

2r2 sin 45◦, P (OCB) =

1

2r2 sin 30◦,

P (OBA) =1

2r2 sin 15◦, P (ODA) =

1

2r2.

Ako je P1 povrsina triju kruznih odjecaka, dobivamo

P1 =r2

4− 1

2r2(sin 45◦ + sin 30◦ + sin 15◦).

Tada je povrsina cetverokuta ABCD (vidi sliku):

P = P (AOB) + P (BOC) + P (COD) − P (ODA),

=r2π

4− r2

2−[r2π

4− 1

2r2(sin 45◦ + sin 30◦ + sin 15◦)

]

=1

2r2(sin 45◦ + sin 30◦ + sin 15◦ − 1)

=1

2r2[(sin 45◦ + sin 15◦) + (sin 30◦ − 1)]

=1

2r2(cos 15◦ − 1

2).

90 PlayMath br. 16-17 (2008.)

Page 91: PlayMath 16-17

πlαy√

matχ

Slika 20.

Ako je Pk povrsina kruga, imamo

Pk

P=

r2π12r2(cos 15◦ − 1

2 )=

cos 15◦ − 12

.

Oznacimo li najmanju stranicu sa a, tada jea

2= r sin 7◦30′, a odatle

r =a

2 sin 7◦30′.

To nam daje

P =a2

8 sin2 7◦30′

(

cos 15◦ − 1

2

)

.

4. (Vidi sliku.) Neka je−−→MA · −−→MB = k. Vrijedi

−−→MA =

−−→MO +

−→OA,

−−→MB =

−−→MO +

−−→OB i

−−→MO = 1

2 (−−→MA +

−−→MB).

To povlaci

k =−−→MA · −−→MB

= (−−→MO +

−→OA)(

−−→MO +

−−→OB)

=−−→MO2 +

−→OA · −−→MO +

−−→MO · −−→OB +

−→OA · −−→OB

=−−→MO2 +

−−→MO(

−→OA +

−−→OB) +

−→OA · −−→OB.

Kako je−→OA = −−−→

OB, slijedi|MO|2 = |OB|2 + k.

Diskusija:

1◦ Ako je konstanta k pozitivna geometrijsko mjesto tocaka je kruznica ciji je polumjer veci od |OB|.2◦ ako je konstanta jednaka 0, geometrijsko mjesto tocaka je kruznica ciji je polumjer jednak |OB|.3◦ Ako je konstanta negativna a po apsolutnoj vrijednosti manja od |OB|2, tada je geometrijsko mjesto tocakakruznica ciji je polumjer manji od |OB|.4◦ Ako je k < −|OB|2 tada ne postoji rjesenje.

5◦ Ako je konstanta k = −|OB|2, geometrijsko mjesto tocaka je kruznica polumjera 0 (tj. tocka O).

http://playmath.petagimnazija.hr 91

Page 92: PlayMath 16-17

πlαy√

matχ

Slika 21.

Slika 22.

5. Vrh pramena je tocka V (x, y) koja lezi na zadanom pravcu za svaku vrijednost od m. Svedemo li zadanu jednadzbuna oblik (x + 2)m = x + y, onda je ona zadovoljena za svaku vrijednost od m, ako je x + 2 = 0 i x + y = 0.Prema tome vrh pramena je (−2, 2).Kako bi pravac p1 prolazio tockom A, mora vrijediti

3 = (m − 1)(−1) + 2m,

a pravac p2 kroz B−1 = (m1 − 1) · 3 + 2m.

Dobivamo da je m = 2, m1 = 25 . Tada je

p1 . . . y = x + 4,

p2 . . . y = −3

5x +

4

5.

Zajednicka je tocka ovih pravaca V (−2, 2). Oznacimo li pravac kroz A i B sa p3, imamo:

p3 . . . y = −x + 2.

92 PlayMath br. 16-17 (2008.)

Page 93: PlayMath 16-17

πlαy√

matχKoeficijenti pravaca su k1 = 1, k2 = − 3

5 , k3 = −1. Dobivamo

∠A = ∠(p1, p2) = 90◦, |AV | =√

2, |AB| = 4√

2, |BV | =√

34.

Takoder vrijedi

tg ϕ =

√2

4√

2=

1

4⇒ ϕ = 14◦.

Simetrala od AB prolazi ishodistem koordinatnog sustava.

IV. razred

1. Ako zadani izrazi cine aritmeticki niz, onda je

sin(y + z − x) + sin(x + y − z) = 2 sin(z + x − y).

Ako se izrazi razviju i srede, dobivamo

2 cosx sin y cos z = sin x cos y cos z + cosx cos y sin z,

sto nakon dijeljenja jednakosti sa cosx cos y cos z, a uz pretpostavku, da je ovaj umnozak razlicit od 0,

2tg y = tg x + tg z.

To znaci da tg x, tg y, tg z cine aritmeticki niz.Svedemo li desnu stranu gornje jednakosti na umnozak, dobiva se

2tg y =sin(x + z)

cosx cos z.

Ako je sada x + z = 90◦, onda je sin(x + z) = 1, cosx = sin z, cos z = sin x, te relacija nakon uklanjanjanazivnika prelazi u

sin 2xtg y = 1 ili sin 2ztgy = 1.

2. Tijelo M ima u trenutku t = 1 apscisu x(1) = 12 − 4 · 1 + 9 = 6. Za t < 1 tijelo miruje pa mu pripadastalna apscisa 6, a to je na dijagramu puta (t, x) prikazano paralelom s osi t. Za t > 1 dijagram puta je parabolax(t) = t2 − 4t + 9, koja ima vrh u tocki t = −−4

2 = 2, x = 5.Brzina tijela je x′(t) = 2t − 4, a njen graf pravac, no tek za t > 1. Za t < 1 tijelo miruje pa je x′(t) = 0.Akceleracija tijela je x′′(t) = 2.Tijelo M ′ pocne se gibati u trenutku t = 0 iz ishodista, pa je x(0) = 0, a isto vrijedi i za t < 0. (Cemu na grafuodgovara negativan dio t-osi. Za t > 0 je x(t) = vt a dijagram puta je pravac, koji prolazi ishodistem a koeficijentsmjera mu je v.Da utvrdimo hoce li se oba tijela i kada sresti, odredimo sjecista grafova njihovih putova, rjesavajuci njihovejednadzbe

x = vt i x(t) = t2 − 4t + 9.

Uvrstavanjem dobiva se jednadzba vt = t2 − 4t + 9, odnosno

t2 − (v + 4)t + 9 = 0,

koja ima rjesenja

t1,2 =v + 4 ±

(v + 4)2 − 36

2.

Diskriminanta D = (v + 4)2 − 36 = 0 za v = 2 ili v = −10.Za v = 2 dobiva se dvostruko rjesenje t = 2+4

2 = 3, a x = 2 · 3 = 6.v = −10 ne dolazi u obzir, jer dobivamo t = −10+4

2 = −3, a u trenutku oba tijela miruju medusobno razmaknuta,pa se ne mogu sresti (vidi sliku). Za v = 2 pravac x = vt tangira parabolu x = t2 − 4t + 9.

http://playmath.petagimnazija.hr 93

Page 94: PlayMath 16-17

πlαy√

matχ

Slika 23.

Iz slike se vidi:Ako je v < 2, ukljucujuci i v = 0 i v < 0, grafovi puta nemaju zajednickih tocaka, tj. tijela se nece sastati.Ako je v = 2, tijela se sastanu samo jednom i to u trenutku t = 3 u udaljenosti x = 6.Ako je v > 2, grafovi imaju dvije zajednicke tocke, tijela se sastaju dva puta i to, ako je 2 < v < 6 oba puta dokse tijelo M giba, a za v ≥ 6 jednom dok M jos miruje a drugi put dok se M giba.

Da bi se tijela srela mora biti v ≥ 2.Ako je v = 9

4 tada se rjesavanjem sustava jednadzbi x = 94 t i x = t2 − 4t + 9 dobiva t1 = 9

4 , t2 = 4 ix1 = x(t1) = 5 1

16 , x2 = x(t2) = 9. Tijela ce se dakle sresti dva puta i to u trenutku t = 94 na mjestu x = 5 1

16 , i utrenutku t = 4 na mjestu x = 9.Srednja brzina tijela M izmedu ova dva susreta ce biti

v =∆x

∆t=

x2 − x1

t2 − t1=

9 − 5 116

4 − 94

=9

4.

3. Iz P (x) = 5x3 + ax3 + bx + c i P (x) + k(x − 1)P ′(x) + (x2 − 1)P ′′(x) = 0, slijedi

(5 + 15k + 30)x3 + (a + 2ak − 15k + 2a)x2 + (b + bk − 2ak − 30)x + (c − bk − 2a) = 0

za sve x ∈ R. Svi koeficijenti moraju biti jednaki 0, pa dobivamo sustav

35 + 15k = 0, a + 2ak − 15k + 2a = 0, b + bk − 2ak = 0, c − bk − 2a = 0.

Rjesenja tog sustava jednadzbi

a = 21, b = 51, c = −77, i k = −7

3.

Znaci P0(x) = 5x3 + 21x2 + 51x − 77. Dobivamo

P0(x)

x − 1= 5x2 + 26x + 77.

Graf funkcije je parabola, kojoj je tjeme u tocki V (−2 35 , 43 1

5 ).

Neka jeP0(x)

x − 1= y = 5x2 + 26x + 77; y′ = 10x + 26, y′(0) = 26.

94 PlayMath br. 16-17 (2008.)

Page 95: PlayMath 16-17

πlαy√

matχ4. Redom dobivamo

f1(x) = ln x f2(x) = 2 lnx f3(x) = 3 ln x,D1(m, ln m) D2(m, 2 ln m) D3(m, 3 ln m),

f ′1(x) =

1

xf ′2(x) =

2

xf ′3(x) =

3

x,

f ′1(m) =

1

mf ′2(m) =

2

mf ′3(m) =

3

m.

Dobivamo da su jednadzbe tangenta redom

t1 . . . y − ln m =1

m(x − m),

t2 . . . y − 2 lnm =2

m(x − m),

t3 . . . y − 3 lnm =3

m(x − m).

Tangente t1 i t2 sijeku se u tocki S(m − m ln m, 0), a tockom S prolazi i tangenta t3 (provjeri!). Dakle sve se tritangente sijeku tocki S. Isto se pokazuje za fr(x) = r ln x, za sve realne brojeve r. Dakle pramen krivulja imasvojstvo da tangente dobivene na opisani nacin prolazi tockom S. Vrh pramena krivulja je fr(x) je tocka (1, 0) jernjom prolaze sve krivulje.Preostaje utvrditi geometrijsko mjesto tocka S. Utvrdimo tok funkcije x(m) = m − m ln m na intervalu 〈0, +∞〉jer m moze poprimiti vrijednosti samo na tom intervalu. Redom dobivamo

x′(m) = 1− ln m − m · 1

m= − ln m,

x′′(m) = − 1

m.

x′(m) = 0 za m = 1, a x′′(1) = −1. Stoga funkcija poprima vrijednost maksimum 1 za m = 1. Kako je

limm→+∞

x(m) = limm→∞

−m(ln m − 1) = −∞

, slijedi da je geometrijsko mjesto tocaka S (jednodimenzionalni) interval 〈−∞, 1] × {0}.

5. Lako se pokaze da x5 i x daju isti ostatak pri dijeljenju sa brojem 5. (Izravnom provjerom ostataka.) Dalje tvrdnjaslijedi lako: (a1 + . . . + an)5 daje isti ostatak pri dijeljenju s 5 kao a1 + . . . + an, a to daje isti ostatak kao ia51 + . . . + a5

n pri dijeljenju s 5.Sve tvrdnje dalje izravno slijede.

http://playmath.petagimnazija.hr 95

Page 96: PlayMath 16-17

πlαy√

matχ

PlayMath

Od raznih aktivnosti ucenika u V. gimnaziji mozda nijedna nije toliko neobicna kao casopis koji upravo drzite urukama. Razne skole imaju razlicite publikacije: skolske listove, godisnjake. . .

Ono po cemu se u pitanju izdavastva V. gimnazija razlikuje je to sto izdaje PlayMath – casopis za matematiku iinformatiku u kojem glavnu rijec imaju ucenici. Ovakav rijetki (vjerojatno i jedini takve vrste) casopis u svijetu V.gimnazija izdaje u suradnji s krovnom udrugom matematicara u Hrvatskoj Hrvatskim matematickim drustvom uzpotporu Ministarstva znanosti, obrazovanja i sporta.

Kako je pocelo?

Prije pojave PlayMath-a, ucenici V. gimnazije su suradivali s raznim matematickim casopisima, radili razne plakate iseminarske radove, pisali maturalne radove iz matematike,. . .

O skolske godine 2001./2002. u V. gimnaziji djeluje grupa koja je proucavala koristenje racunala u matematici. Takosu ucenici stekli znanja o radu crtanju slika i procavanju geometrije programom dinamicne geometrije (The Geometer’sSketchpad), te o koristenju programa za simbolicku algebru1 (MAPLE).

Dobar tih zadataka i seminara trebalo je oblikovati u neki smislen matematicki tekst. Doslo je do potrebe da se zapisematematicki tekst u nekom prikladnom racunalnom formatu, te da se u njemu predstavi rad napravljen na racunalu. Tadasu se mnogi ucenici prvi puta sreli s LATEX-om2 koji im je omogucio lakse pisanje (slozenog) matematickog teksta ijednostavniju medusobnu (matematicku) komunikaciju.

Skolske godine 2002./2003. u tadasnjem 3.f razredu V. gimnazije doslo je do inicijative za pokretanje matematickogcasopisa. Razred je odlucio pocet s radom na ovom zanimljivom i izazovnom projektu.

0. urednistvo i prva 4 broja

Prvo je trebalo izabrati ime za casopis. Iz prvog reda do prozora je netko doviknuo neka se casopis zove PlayMath.Odluceno je da ce privremeno to biti radni naziv. Kako se kasnije nismo uspjeli dogovoriti oko drugog naziva radni nazivje ostao.

Smisljen je popis tema za koje su kasnije podijeljene ucenicima u razredu sukladno njihovim sklonostima (jedni suradili anketu, drugi su pisali o povijesnim temama, treci o knjizi vezanoj uz matematiku, cetvrti crtali strip,. . . ).

Rad na prvom broju je poceo u listopadu 2002. godine Pocetkom iduce godine osvanuo je prvi broj PlayMath-a,a nedugo nakon toga na web-adresi http://www.petagimnazija.hr/math i prvo web-izdanje casopisa. Namatematickom klubu odrzanom u XV. gimnaziji u organizaciji HMD-a predstavljen je prvi broj casopisa.

Prvi broj smo uspjeli tiskati zahvaljujuci sponzorima koji su podrzali ovu neobicnu inicijativu.

Te skolske godine 2002./2003. dosao je poziva da se prijave razni ucenicki radovi na LIDRANO (smotru literarnog,dramskog i novinarskog stvaralastva, medu njima i smotra skolskih casopisa). Nismo bili sigurni spada li PlayMath naLidrano, ali nakon sto smo proucili propozicije odlucili smo ga poslati. Kako smo prosli citajte u [1].

Potaknuti uspjehom 1. broja koji je dobro primljen, pocetkom svibnja izasao je 2. broj koji je predstavljen na drzavnomnatjecanju iz matematike.

U skolskoj godini 2003./2004. odluceno je da PlayMath nastavi izlaziti. Te godine na natjecaju Ministarstva pros-vjete i sporta za poticanje izvannastavnog rada s ucenicima V. gimnazija je dobila financijsku podrsku za ovaj projekt. Naslicnom natjecaju i grad Zagreb dao je financijsku podrsku ovom projektu. Time olaksano dalje izlazenje PlayMath-a.

Urednistvo je odradilo svoj posao i nastala su jos dva nova broja.

Prva 4 broja PlayMath jos nije bio sluzbeno registriran, pa se to razdoblje smatra kao 0. urednistvo. Clanovi urednistvasu bili: Tvrtko Tadic (IZVRSNI UREDNIK), Marko Horvat (ODGOVORNI UREDNIK, predlagac pokretanja casopisa),

1engl. CAS – Computer Algebra Software2O LATEX-u imali ste prilike citati u PlayMath-u br. 2(2003.)

96 PlayMath br. 16-17 (2008.)

Page 97: PlayMath 16-17

πlαy√

matχ

Slika 1. Prva naslovnica i prvo web-izdanje

Karla Lemac (GRAFICKA UREDNICA do broja 2), Dino Malpera (UREDNIK WEB-IZDANJA do 3. broja, GRAFICKIUREDNIK 3. broja), Filip Niksic (CLAN UREDNICKOG SAVJETA, UREDNIK WEB-IZDANJA 4. BROJA), Daria Po-povic (CLANICA UREDNICKOG SAVJETA, GRAFICKA UREDNICA od 4. broja), Tomislav Herman, Martina Nimac, TiaTomisa, Ana Virag (CLANOVI UREDNICKOG SAVJETA), Matija Basic, Iva Kelava, Valentin Lapaine, Hana Severdija(CLANOVI UREDNICKOG SAVJETA od 3. broja), Vanja Vagner, Hrvoje Torbasinovic (CLANOVI UREDNICKOG SA-VJETA od 4. broja).

U ovom razdoblju PlayMath je objavio preko 45 clanaka i priloga na 248 stranica (od kojih izdavajamo: Hiperprostor,Pet rjesenja geometrijskog zadatka, Igra SET i matematika vezana uz nju, Donald Ervin Knuth, Zlatni rez, D’Hondtovametoda raspodjele, Zbroji 15, Perceptivne varke, Olimpijadama oko svijeta, Uvrsti i sredi, Tko je pobjednik?, Najvececijelo bxc i njegovi prijatelji, Model sata, Analiza ankete provedene medu sudionicima XII. drzavnog natjecanja iz ma-tematike, Vicevi. . . ) u nekoliko rubrika (clanci, matematicki software, veliki matematicari, dogadanja, zadatci irjesenja, ankete, matematicka igraonica, knjige, zabavni kutak, matematika i demokracija, prijemni ispiti, mate-matika i Internet, razgovori).

Kad se raspravljalo o tome kako urediti casopis, urednistvo je zeljelo da svaki clanak bude oznacen nekakvim orna-mentima. Kako u pocetku nismo znali sami izradivati ornamente, posluzili s mo se gotovim rjesenjima prof. dr. sc. DarkaZubrinica3 tzv. pleter

���������i stecak � � � fontovima.

3Profesor na Zavodu za primijenjenu matematiku, FER, Zagreb.

http://playmath.petagimnazija.hr 97

Page 98: PlayMath 16-17

πlαy√

matχKrajem skolske godine 2003./2004. postavilo se pitanje: Sto dalje? Prvo je odlukom skolskog odbora V. gimnazije

sluzbeno osnovan casopis PlayMath. Nakon cega se pristupilo registraciji casopisa.

1. urednistvo i iducih 8 brojeva

Prilikom osnutka casopisa donesena je odluka da ce PlayMath izlaziti 3 puta godisnje. Kako bi se osigurala stabilnostcasopisa urednistvo ce ubuduce ciniti i studenti (a ne samo ucenici kao do tada). Urednistvu ce na raspolaganju biti istrucni suradnici. U projekt izdavanja casopisa ukljucilo se i Hrvatsko matematicko drustvo. Od 5. broja casopis je usustavu financiranja Ministarstva znanosti, obrazovanja i sporta4.

Kako bi se osigurao dalji razvoj prisutnosti PlayMath-a na Internetu, od broja 6. web-izdanje PlayMath-a djeluje nanovoj adresi http://playmath.skolstvo.t-com.hr. Web-izdanje postaje portal s vijestima iz svijeta matema-tike, a od 7. broja na njemu djeluje i Forum podmlatka HMD-a. Web-izdanje PlayMath-a postaje jedna od najposjecenijihmatematickih stranica u Hrvatskoj.

Slika 2. Naslovnica 6. broja i drugo web-izdanje

U ovom razdoblju PlayMath je nastavio redovito izlaziti. Svaki broj je donosio preko 15 novih clanaka i priloga pobroju (od kojih izdavajamo: Jednadzba skoljke, Profesor sa stranica Men’s Health-a, Evolucija covjeka – biologija ilimatematika, Dinamicko programiranje, Napoleon i njegovo vrijeme, Borba covjeka i racunala na 64 polja, Znanstvenicas vjetrom u ledima, Predstava Dokaz, Radikalno srediste, Ada Augusta Byron, Sto mislite o informatici?, Odredivanjegranicne crte u zaljevu, . . . i to je Amerika, Zivot izmedu dva kvadrata, Vicevi. . . ). Ustroj rubrika ostao je isti uz pokojunovu rubriku i prosirenje postojecih.

Zbog ucestalih pojavljivanja oznaka za smileice (smjesko):) poceo se koristiti font smile ����� , a kasnijim usvrsavanjemfonta dobiveni su i veci oblici

�����. Neki obilci ornamenata koje PlayMath koristi od 1. broja su doradeni. Tako je

dobiven novi font tzv. stecA dobiven iz starog oblika�����

. Brojevi 10.-12. izasli su na smanjenomformatu A4, dok su svi prethodni brojevi izlazili na formatu A4.

Urednistvo je radilo u sastavu: Tvrtko Tadic (GLAVNI UREDNIK), Dijana Kreso (POMOCNICA GL. UREDNIKA od5. do 8. broja, UREDNICA od 9. do 12.) Rudi Mrazovic (POMOCNIK GL. UREDNIKA od 6. do 8. broja, UREDNIK 5. iod 6. do 11. broja), Filip Niksic (UREDNIK WEB-IZDANJA), Daria Popovic (GRAFICKA UREDNICA), Nikola Grubisic(UREDNIK od 5. do 11. broja), Kristina Skreb (UREDNICA od 5. do 10. broja, CLANICA UREDNICKOG ODBORA11. broja), Andela Zaric (UREDNICA od 5 do 8. broja), Ana Virag (UREDNICA od 5. do 7. broja), Hana Severdija(UREDNICA od 5. do 7. broja, CLANICA UREDNICKOG ODBORA od 8. do 12. broja), Tia Tomisa (UREDNICA 5. brojai CLANICA UREDNICKOG ODBORA od 6. do 7. broja), Matija Basic (CLAN UREDNICKOG ODBORA od 5. do 8. broja,UREDNIK od 9. do 12. broja), Filip Lavriv (CLAN UREDNICKOG ODBORA od 5. do 6. broja, UREDNIK od 7. do 12.broja), Marina Sijak (CLANICA UREDNICKOG ODBORA od 5. do 9. broja), Vanja Vagner (CLANICA UREDNICKOGODBORA od 5. do 12. broja), Martina Nimac (CLANICA UREDNICKOG ODBORA od 5. do 8. broja), Dino Malpera

4Novo ministarstvo nastalo spajanjem nekadasnjeg Minstarstva prosvjete i sporta i Ministarstva znanosti i tehnologije.

98 PlayMath br. 16-17 (2008.)

Page 99: PlayMath 16-17

πlαy√

matχ

Slika 3. Naslovnica 12. broja i Forum podmlatka HMD-a

(CLAN UREDNICKOG ODBORA od 5. do 7. broja), Marina Sliskovic (CLANICA UREDNICKOG ODBORA od 5. do 7.broja, UREDNICA od 8. do 12. broja), Ivo Sluganovic (CLAN UREDNICKOG ODBORA od 5. do 7. broja, POMOCNIKGL. UREDNIKA od 9. do 11. broja, UREDNIK 5. i 12. broja), Ivan Gavran (CLAN UREDNICKOG ODBORA od 6. do 7.broja, POMOCNIK GL. UREDNIKA od 9. do 11. broja, UREDNIK 5. i 12. broja), Petar Mrazovic (CLAN UREDNICKOGODBORA od 6. do 12. broja), Goran Zuzic (CLAN UREDNICKOG ODBORA od 8. do 12. broja), Viktorija Sukser(CLANICA UREDNICKOG ODBORA od 9. do 12. broja), Adrian Satja Kurdija (UREDNIK od 11. do 12. broja).

Mnogi (stariji) clanovi urednistva, zbog svojih brojnih obaveza, vise nisu mogli ispunjavati obaveze prema casopisu.Stoga je odluceno da je vrijeme za smjenu generacije i da nova ekipa preuzme PlayMath.

2. urednistvo

Nakon dugogodisnjeg urednika Tvrtka Tadica, vodenje PlayMath-a je preuzeo Adrian Satja Kurdija.

Slika 4. Trece web-izdanje

Mnogi novi ucenici skole pristupili su urednistvu i napisali brojne zanimljive clanke koji su ispunili brojeve V. godistaPlayMath-a (brojevi 13, 14, 15). Objavljeno je ukupno 60-ak clanaka i priloga na ukupno 198 stranica, od kojih izdvajamo:Cezarova sifra, Matematicki i logicki paradoksi, Vjerojatnosti i problem rodendana, Matematika u prirodi, Neocekivaneprimjene A-G nejednakosti, Eulerova cigla, Rubikova kocka, Hanojski tornjevi, Nashov ekvilibrij, Milenijski problemi,John Forbes Nash, Invarijante, Racunalo grijesi u racunu, Glava ili pismo, Courntov model duopola, Matematika uumjetnosti, Vedska matematika, Vicevi. . . Rubrike pod kojima su clanci i prilozi izlazili uglavnom su slijedile tradicijubivseg urednistva.

http://playmath.petagimnazija.hr 99

Page 100: PlayMath 16-17

πlαy√

matχNovo urednistvo donijelo je i mnoge novine. Naslovnice casopisa su drugacije nego prije, a svaka od njih umjetnicko

je djelo grafickog urednika Vilima Stiha, koji je dizajnirao i novi PlayMath-ov logo.

Nadalje, obnovljen je izgled naseg web-izdanja, koje ima i novu adresu

http://playmath.petagimnazija.hr/,

a posebno smo ponosni na modernizirani Forum podmlatka HMD-a

http://playmath.petagimnazija.hr/forum/

koji sada ima razne mogucnosti koje prije nije imao, kao sto su ankete, mogucnost pisanja matematickog teksta, privatneporuke...

Slika 5. Modernizirani Forum podmlatka HMD-a

Za sve to odgovoran je web urednik Tomislav Gudlek. Takoder, na forumu su organizirana i dva kola Online natjecanjana kojima su sudjelovali ucenici osnovnih i srednjih skola.

Kakvi su planovi za buducnost? Namjeravamo sadrzajno obnoviti PlayMath-ovo web izdanje, na kojem se zasadasamo objavljuju matematicke novosti iz Hrvatske i svijeta. Prikupljamo nove ideje koje bi poboljsale izgled i sadrzajcasopisa, a bili bismo posebno zadovoljni kada bi se vise citatelja ukljucilo u pisanje clanaka / priloga ili u uredivanjePlayMath-a. Zato pozivamo sve zainteresirane na suradnju.

Dodatne informacije[1] Severdija H., LIDRANO, PlayMath 2 (2003.), str. 39-40.

[2] Gusic L., PLAYMATH (intervju s urednistvom), Matka br. 48 (2004.)

[3] Tadic T., PLAYMATH, Matka br. 47 (2003.)

[4] Mladinic P., CASOPIS PLAYMATH, Poucak br. 13 (2003.)

Abstract

PlayMath was started in the school year 2002./2003. by a group of students. In that first year two issues werepublished. And in the school year 2003./2004. two new issues were published. In the school year the concept of themagazine changed. Since then the editorial board consists of high school and university students and the magazine ispublished three times a year. All the time PlayMath has had its web edition on which the editorial board tried to be morethan just a web page on the magazine. The contents of the web edition have changed over the years. First it was a webpage with information about the magazine, contents, article abstracts, and some additional material. Later it started topublish news and have sections with different contents. In 2005. The Forum of Young Mathematicians was started on theweb edition. Soon the web edition became one of the most visited mathematical web pages in Croatia.

The new editorial board is hoping to bring new people to the magazine and refresh its contents.

100 PlayMath br. 16-17 (2008.)